How does light 'choose' between wave and particle behaviour?Is the wave-particle duality a real duality?Does electron in wave form have mass?Local EPR-experiments with photons in vacuum?Can a single photon behave in a non-local way?Is light particle of wave?If matter and light have dual-nature, shouldn't we able to explain the observed phenomenons using either wave or particle?Have they really photographed light behaving both as a particle and a wave?Why does a real/virtual photon interact only with charged particle?How does wave-particle duality describe Photoelectric effect?Why wave theory cannot explain photoelectric effect and provides evidence for particle nature of light?Dual nature of matter and radiationHow to decide whether light shows particle of wave nature?Relationship Between Saturation Current and Light Intensity According to Wave Theory of LightQuantization and wave-particle dualism of light

Why do games have consumables?

Philosophical question on logistic regression: why isn't the optimal threshold value trained?

Is Diceware more secure than a long passphrase?

Find a stone which is not the lightest one

How to pronounce 'c++' in Spanish

Mistake in years of experience in resume?

How can I wire a 9-position switch so that each position turns on one more LED than the one before?

Nails holding drywall

Check if a string is entirely made of the same substring

Do I need to watch Ant-Man and the Wasp and Captain Marvel before watching Avengers: Endgame?

Why doesn't the standard consider a template constructor as a copy constructor?

Why did Rep. Omar conclude her criticism of US troops with the phrase "NotTodaySatan"?

How exactly does Hawking radiation decrease the mass of black holes?

How to keep bees out of canned beverages?

Is there any pythonic way to find average of specific tuple elements in array?

A ​Note ​on ​N!

What is purpose of DB Browser(dbbrowser.aspx) under admin tool?

Can someone publish a story that happened to you?

How long after the last departure shall the airport stay open for an emergency return?

Why must Chinese maps be obfuscated?

Retract an already submitted recommendation letter (written for an undergrad student)

Negative Resistance

Magical attacks and overcoming damage resistance

"The cow" OR "a cow" OR "cows" in this context



How does light 'choose' between wave and particle behaviour?


Is the wave-particle duality a real duality?Does electron in wave form have mass?Local EPR-experiments with photons in vacuum?Can a single photon behave in a non-local way?Is light particle of wave?If matter and light have dual-nature, shouldn't we able to explain the observed phenomenons using either wave or particle?Have they really photographed light behaving both as a particle and a wave?Why does a real/virtual photon interact only with charged particle?How does wave-particle duality describe Photoelectric effect?Why wave theory cannot explain photoelectric effect and provides evidence for particle nature of light?Dual nature of matter and radiationHow to decide whether light shows particle of wave nature?Relationship Between Saturation Current and Light Intensity According to Wave Theory of LightQuantization and wave-particle dualism of light













54












$begingroup$


Light exhibits wave behaviour in phenomenon such as interference but particle behaviour in the photoelectric effect. How does light 'choose' where to be a wave and where to be a particle?










share|cite|improve this question









$endgroup$











  • $begingroup$
    This question is briefly addressed in the last paragraph of this answer: physics.stackexchange.com/a/469623
    $endgroup$
    – Chiral Anomaly
    Apr 19 at 1:44










  • $begingroup$
    Related questions on this confusion about what we actually mean by wave-particle duality: physics.stackexchange.com/q/46237/50583, physics.stackexchange.com/q/100443/50583
    $endgroup$
    – ACuriousMind
    Apr 19 at 9:16






  • 1




    $begingroup$
    Light doesn't behave like "a particle and/or a wave": light behaves according to the Maxwell's equations (and their quantisation) and wave vs particle is an old unfortunate terminology.
    $endgroup$
    – gented
    Apr 21 at 11:24















54












$begingroup$


Light exhibits wave behaviour in phenomenon such as interference but particle behaviour in the photoelectric effect. How does light 'choose' where to be a wave and where to be a particle?










share|cite|improve this question









$endgroup$











  • $begingroup$
    This question is briefly addressed in the last paragraph of this answer: physics.stackexchange.com/a/469623
    $endgroup$
    – Chiral Anomaly
    Apr 19 at 1:44










  • $begingroup$
    Related questions on this confusion about what we actually mean by wave-particle duality: physics.stackexchange.com/q/46237/50583, physics.stackexchange.com/q/100443/50583
    $endgroup$
    – ACuriousMind
    Apr 19 at 9:16






  • 1




    $begingroup$
    Light doesn't behave like "a particle and/or a wave": light behaves according to the Maxwell's equations (and their quantisation) and wave vs particle is an old unfortunate terminology.
    $endgroup$
    – gented
    Apr 21 at 11:24













54












54








54


10



$begingroup$


Light exhibits wave behaviour in phenomenon such as interference but particle behaviour in the photoelectric effect. How does light 'choose' where to be a wave and where to be a particle?










share|cite|improve this question









$endgroup$




Light exhibits wave behaviour in phenomenon such as interference but particle behaviour in the photoelectric effect. How does light 'choose' where to be a wave and where to be a particle?







quantum-mechanics wave-particle-duality






share|cite|improve this question













share|cite|improve this question











share|cite|improve this question




share|cite|improve this question










asked Apr 18 at 15:05









d_gd_g

42127




42127











  • $begingroup$
    This question is briefly addressed in the last paragraph of this answer: physics.stackexchange.com/a/469623
    $endgroup$
    – Chiral Anomaly
    Apr 19 at 1:44










  • $begingroup$
    Related questions on this confusion about what we actually mean by wave-particle duality: physics.stackexchange.com/q/46237/50583, physics.stackexchange.com/q/100443/50583
    $endgroup$
    – ACuriousMind
    Apr 19 at 9:16






  • 1




    $begingroup$
    Light doesn't behave like "a particle and/or a wave": light behaves according to the Maxwell's equations (and their quantisation) and wave vs particle is an old unfortunate terminology.
    $endgroup$
    – gented
    Apr 21 at 11:24
















  • $begingroup$
    This question is briefly addressed in the last paragraph of this answer: physics.stackexchange.com/a/469623
    $endgroup$
    – Chiral Anomaly
    Apr 19 at 1:44










  • $begingroup$
    Related questions on this confusion about what we actually mean by wave-particle duality: physics.stackexchange.com/q/46237/50583, physics.stackexchange.com/q/100443/50583
    $endgroup$
    – ACuriousMind
    Apr 19 at 9:16






  • 1




    $begingroup$
    Light doesn't behave like "a particle and/or a wave": light behaves according to the Maxwell's equations (and their quantisation) and wave vs particle is an old unfortunate terminology.
    $endgroup$
    – gented
    Apr 21 at 11:24















$begingroup$
This question is briefly addressed in the last paragraph of this answer: physics.stackexchange.com/a/469623
$endgroup$
– Chiral Anomaly
Apr 19 at 1:44




$begingroup$
This question is briefly addressed in the last paragraph of this answer: physics.stackexchange.com/a/469623
$endgroup$
– Chiral Anomaly
Apr 19 at 1:44












$begingroup$
Related questions on this confusion about what we actually mean by wave-particle duality: physics.stackexchange.com/q/46237/50583, physics.stackexchange.com/q/100443/50583
$endgroup$
– ACuriousMind
Apr 19 at 9:16




$begingroup$
Related questions on this confusion about what we actually mean by wave-particle duality: physics.stackexchange.com/q/46237/50583, physics.stackexchange.com/q/100443/50583
$endgroup$
– ACuriousMind
Apr 19 at 9:16




1




1




$begingroup$
Light doesn't behave like "a particle and/or a wave": light behaves according to the Maxwell's equations (and their quantisation) and wave vs particle is an old unfortunate terminology.
$endgroup$
– gented
Apr 21 at 11:24




$begingroup$
Light doesn't behave like "a particle and/or a wave": light behaves according to the Maxwell's equations (and their quantisation) and wave vs particle is an old unfortunate terminology.
$endgroup$
– gented
Apr 21 at 11:24










11 Answers
11






active

oldest

votes


















134












$begingroup$

In fact, light is not really a wave or a particle. It is what it is; it's this strange thing that we model as a wave or a particle in order to make sense of its behaviour, depending on the scenario of interest.



At the end of the day, it's the same story with all theories in physics. Planets don't "choose" to follow Newtonian mechanics or general relativity. Instead, we can model their motion as Newtonian if we want to calculate something like where Mars will be in 2 weeks, but need to use general relativity if we want to explain why the atomic clock on a satellite runs slow compared to one on the ground.



Light doesn't "choose" to be a wave or a particle. Instead, we model it as a wave when we want to explain (or calculate) interference, but need to model it as a particle when we want to explain (or calculate) the photoelectric effect.






share|cite|improve this answer











$endgroup$












  • $begingroup$
    Comments are not for extended discussion; this conversation has been moved to chat.
    $endgroup$
    – ACuriousMind
    Apr 22 at 10:09


















17












$begingroup$


How does light 'choose' where to be a wave and where to be a particle?




It doesn't, YOU do. That's really the entire "weirdness" of quantum right there.



It's not entirely crazy. If you measure a car with a scale it will tell you it's 1200 kg, and if you measure it with a spectrometer it will tell you its red. This is perfectly natural.



The thing that makes quantum weird is that you can measure the same thing twice and get two different answers. More weird, some of those measurements are linked to each other so if you measure one the other changes.



Its as if you measured the weight of your car and the length changed. And then you measured the length and the weight changed. This precise thing happens in quantum, for instance, the position and momentum of a particle are linked in this way



In any event, the wave-or-particle nature is entirely up to you. Which nature you see is based on the experiment you use, not the photon itself.



The idea that something you do has this "radical" effect on the outcome is what drives everyone mad in QM.






share|cite|improve this answer











$endgroup$








  • 1




    $begingroup$
    This could be a nice answer if you could elaborate a bit.
    $endgroup$
    – Gert
    Apr 18 at 15:11










  • $begingroup$
    If you can choose the state of light (for eg), would that mean that theoretically we could influence light to behave as a wave and not as photons in phenomenon such as the photoelectric effect?
    $endgroup$
    – d_g
    Apr 18 at 15:15






  • 12




    $begingroup$
    Ahhh but there you go... how do you measure the photoelectric effect? By measuring its energy. Presto, it looks like a particle. Take that exact same photon and put it in a double-slit? Presto, looks like a wave. But these terms, "particle" and "wave" are the real problem, you're using classical terms that simply don't describe what's really happening. You would have a similar difficulty trying to describe a car to someone who only saw horses and tried to say the wheels were sort of like legs... how often do you need new horseshoes?
    $endgroup$
    – Maury Markowitz
    Apr 18 at 15:18






  • 1




    $begingroup$
    If you measure the weight of car with a beam scale first and then with a spring scale, you'll get the same weight twice but in the second case your car may stretch itself under its own weight. So even in "macro" the way of measuring can have an impact on the properties of the object observed.
    $endgroup$
    – Echox
    Apr 18 at 20:33






  • 2




    $begingroup$
    @MauryMarkowitz you need new horseshoes twice per year, once to make your horse not slip on ice, once to make him eat less while running on not-ice.
    $endgroup$
    – John Dvorak
    Apr 19 at 12:48


















12












$begingroup$

Light always behaves as a wave. Particles can be thought of as a combination of waves, a wave packet. What determines which behavior you'll get is the length scale of the system the light is interacting with and the wavelength of the impinging light.



Suppose you have a series of Bowling Balls suspended in the air making a wall, but with spaces between. Shoot a bunch of 2mm diameter ball bearings (BBs) at the bowling balls. Some will pass through the wall, hitting no bowling balls. Some will hit a ball and bounce in some direction.



Pay close attention to where the BBs come from and their initial speed and where they end up, and you will be able to tell the shape, size, and position of the bowling balls.



Reverse the problem. Have a bunch of suspended BBs in the air and shoot bowling balls at them. Each bowling ball will hit multiple BBs. The outgoing trajectory won't tell you much about the BBs.



Strike a tuning fork, it vibrates, making a characteristic sound. Play that note at high volume, you can set the tuning fork to vibrating. It doesn't start to vibrate at just any wavelength.



To a particle, you can associate the De Broglie wavelength, $h/p$, where $p$ is the momentum. The higher the momentum, the lower the wavelength, the more particle like. Whereas macroscopic but small openings can be used with light in the double slit experiment, you need electron crystallography to demonstrate the same effects with electrons: Electron diffraction



If you have a large wavelength of light, it will interact with multiple particles of a system depending on that system. If the wavelength is sufficiently small, and so the energy sufficiently high, it can interact with a single electron instead of multiple electrons, imparting all its energy to an electron and resulting in the photoelectric effect, a particle-particle scattering effect. Change the wave length and the interaction takes on a more classical form.



In addition to the wavelengths involved you want to pay attention to the number of photons available. The fewer the photons interacting with the system, the more quantum-like it is. The higher the density of photons, the more classical the light will behave. For a more detailed explanation on the barrier between quantum vs. classical behavior, see the intro and first chapter of Griffith's text on electromagnetism.



In short, the behavior you get depends on the De Broglie wavelength of the light/particle, how many particles are inbound and how the lengths scales of the target compare with the De Broglie wavelength.






share|cite|improve this answer











$endgroup$












  • $begingroup$
    Any mathematical account of the correspondence principle will also give insight into how classical effects emerge in a limit from quantum ones.
    $endgroup$
    – J.G.
    Apr 18 at 19:14






  • 2




    $begingroup$
    @R. Romero did you mean baseballs or ball bearings when you said "BBs" in your original answer?
    $endgroup$
    – binaryfunt
    Apr 18 at 20:13










  • $begingroup$
    Ball bearings. sorry, I should have been clear. Tiny ball bearings a few cm in diameter.
    $endgroup$
    – R. Romero
    Apr 18 at 20:20






  • 1




    $begingroup$
    -1 this answer is completely wrong, I don't understand why it gets so many upvotes: "Particles can be thought of as a combination of waves, a wave packet" is wrong. A wave packet is a completely different thing; moreover there are no photons in QM (they emerge in QFT only). Last but not the least, the de Broglie wavelength has nothing to do with the "wave nature". I don't want to sound rude but this answer is a salad of physics misconceptions and should be deeply amended.
    $endgroup$
    – gented
    Apr 21 at 11:27







  • 1




    $begingroup$
    Einstein's paper doesn't demonstrate quanta and Bohr's models aren't the way quantum physics is formulated: they were initial ad hoc models to make things work in a certain way. Quantum mechanics and quantum field theory have been formalised since then and all those quantities (photons, quanta and all the rest) have a precise meaning which conceptually isn't what you have described.
    $endgroup$
    – gented
    2 days ago



















8












$begingroup$


How does light 'choose' where to be a wave and where to be a particle?




It doesn't. It always behaves as a wave (obeying the principle of superposition), and it always behaves as a particle (particle number being quantized).



It sounds like you may have been influenced by someone who told you that light behaves like a particle in some experiments, and like a wave in others. That's false.






share|cite|improve this answer









$endgroup$








  • 4




    $begingroup$
    How do you address the fact that the photoelectric effect doesn't make sense if you consider light as a wave though? I don't think it is fine to say it always behaves like a wave, unless you mean to say the only consistent wave behavior is superposition
    $endgroup$
    – Aaron Stevens
    Apr 18 at 15:40










  • $begingroup$
    @AaronStevens The photoelectric effect doesn't "disprove" a wave description. If I place a guitar on a table and dust settles on one string, which I then pluck, whether I displace no specks of dust, one of them or several depends on how hard I pluck. But what caused any such displacement? A wave transmitting energy down the string. It's just that in QM, this is also explicable in terms of individual photons' absorption. This is why I recommend learning QFT, which makes better sense of how the wave/particle aspects mesh together.
    $endgroup$
    – J.G.
    Apr 18 at 19:19










  • $begingroup$
    @AaronStevens: How do you address the fact that the photoelectric effect doesn't make sense if you consider light as a wave though? Could you explain why you think this?
    $endgroup$
    – Ben Crowell
    Apr 18 at 21:48










  • $begingroup$
    @BenCrowell I agree that QFT resolves this. I was talking more along the lines of classic EM waves where you would expect the intensity of the light to influence the energy of the ejected electrons and the frequency to influence the rate at which electrons are ejected, yet we see the reverse of this.
    $endgroup$
    – Aaron Stevens
    Apr 18 at 22:44











  • $begingroup$
    I'm guessing this is where the OP's confusion lies. We have some phenomena where the wave picture of light is sufficient to explain what's going on, but other times it doesn't work at all. When you are first learning it all it seems rather arbitrary without any deeper structure, where you just have to remember which picture is right to use depending on the system.
    $endgroup$
    – Aaron Stevens
    Apr 18 at 22:49


















5












$begingroup$

Light always behaves as a particle and waves .So there is no particular time when it can behaves like a particle but not wave and vice versa .Thus light carries both of these two nature(particle and wave) along with it (light)for all time.
Actually why is my thinking is like light has two of these nature for all time?



Reasons for considering it as *wave *



  1. James Clerk Maxwell proved that speed of the electromagnetic waves in free space is the same as the speed of light c=2.998 ×10^8m/s. Thereby he concluded that light consists of electromagnetic waves .So it has wave nature for all time.

  2. Light also shows *interference *,*diffraction *,*polarization *.
    And all of these property show that light has wave nature and it is for all time.

Reasons for considering light as particle



  1. Max plank in 1900 introduced concept of quantum of energy .The energy exchange between radiation and surroundings is in *discrete or *quantised * form i.e energy exchange between electromagnetic waves is integer multiple of (plank constant *frequency of wave ).As light consists of electromagnetic waves then light is made of discrete bits of energy i.e photons with energy (plank constant *frequency of light).
    Thereby photoelectric effect ,Compton effect all show the particle nature of light.

So radiation (electromagnetic waves) exhibit particle nature conversely material particles display wave like nature introduced by de Broglie(a moving particle has wave properties associated with it).This is confirmed experimentally by Davisson and Germer they proved that interference which is a property of waves can be obtained by with material particles such as electron.



So as radiation exhibit particle like nature but particles also exhibit
wave- like behavior .Thereby light always shows two type of natures both as waves and particles .






share|cite|improve this answer









$endgroup$












  • $begingroup$
    Ok if we say light is a particle and wave at all times, then why does it exhibit wave properties in certain cases and particle behaviour in certain phenomenon? Is it because it acts more like a wave and more like a particle in certain conditions? If so what are those conditions? Appreciate your answer btw
    $endgroup$
    – d_g
    Apr 18 at 17:58







  • 1




    $begingroup$
    @d_g light as a whole shows two natures as i said. you can consider double silt experiment when light passes through the silt it behaves like a wave but when it strikes the screen it behaves like a particle .
    $endgroup$
    – Sneha Banerjee
    Apr 18 at 19:04










  • $begingroup$
    Sounds good at 1st glance (as do the same general assertions from other respondents) BUT the single/double slit example can see "observed behaviour" change between apparent modes when the slit is opened/closed with timing such that behaviour "change" occurs with information transfer faster than C. ie a photon (or eg electron etc) ALWAYS "knows" whether the 'slit B' is open or closed at the time that it transits slit A and/or when it arrives at the target surface, even when the time taken travelling at C from slit B to slit A or target is > than the time that the photon etc takes to arrive.
    $endgroup$
    – Russell McMahon
    Apr 21 at 2:43


















4












$begingroup$

The fundamental experiment showing the apparent contradiction is Young's double slit: How can particle characteristics be transmitted when there is only an interfering wave between the point A of emission and point B of absorption?



However, for photons in vacuum (moving at c) there is a simple answer: The spacetime interval between A and B is empty, it is zero! That means that both points A and B are adjacent. A and B may be represented by mass particles (electrons etc.) which are exchanging a momentum. The transmission is direct, without need of any intermediate particle.



In contrast, a spacetime interval cannot be observed by observers. If a light ray is transmitted from Sun to Earth, nobody will see that A (Sun) and B (Earth) are adjacent. Instead, they will observe a space distance of eight light minutes and a time interval of eight minutes, even if the spacetime interval is zero. In this situation, the light wave takes the role of a sort of "placeholder": Light waves are observed to propagate at c (according to the second postulate of special relativity), but this is mere observation.



In short, the particle characteristics may be transmitted without any photon because the spacetime interval is zero. The wave characteristics (including the propagation at c) are observation only.



By the way, for light propagating at a lower speed than c (e.g. light moving through a medium), we need quantum mechanics for the answer.






share|cite|improve this answer









$endgroup$












  • $begingroup$
    Double slit works for eg electrons.
    $endgroup$
    – Russell McMahon
    Apr 20 at 21:29










  • $begingroup$
    I like you acknowledgement that all photons huddle together in a timeless spaceless group nattering together (presumably) and waiting the end of all things :-). I've long loved and pondered this aspect of reality and wondered if we can ever properly appreciate its meaning :-). || I wrote my one liner above just before rushing off to Easter Sunday church service. I've just reread it at further below write-speed and note now your assertions relating to A & B being able to be massed particles but the communications being mediated not at C but instantaneously ...
    $endgroup$
    – Russell McMahon
    Apr 21 at 2:33










  • $begingroup$
    ... (or at least > or >> C, as is seen to be the case experimentally). I don't recall seeing that asserted anywhere previously (this is not a specialist area of mine). It doesn't SEEM right but it certainly makes sense of the results :-). Do you have any references for that?
    $endgroup$
    – Russell McMahon
    Apr 21 at 2:35










  • $begingroup$
    @Russell McMahon, the transmission of the momentum is not instantaneous, in the Sun-Earth example 8 minutes, and the distance is still 8 light minutes. I simply emphasize the fact that the spacetime interval is zero, and that means that from this point of view A and B are adjacent such that there is a possibility for transmission of particle characteristics. In contrast, quantum mechanics is needed when mass is concerned (light through medium, wave-particle duality of mass particles such as electrons).
    $endgroup$
    – Moonraker
    Apr 21 at 5:37











  • $begingroup$
    You might also be interested in this question which did not receive a satisfying answer yet. Happy Easter!
    $endgroup$
    – Moonraker
    Apr 21 at 5:39


















4












$begingroup$

The light doesn't chose. You as an experimenter chose which observable you want to measure, and thus which operator you use. Such measurement will result in a wavefunction collapsing into one of the eigenstates of this operator.
E.g. a positiin operator will give you a position, i.e. particle.
A momentum operator will give you a momentum, i.e. wavelike object.






share|cite|improve this answer








New contributor




Alex Vozny is a new contributor to this site. Take care in asking for clarification, commenting, and answering.
Check out our Code of Conduct.






$endgroup$












  • $begingroup$
    No. See my comment to @Sneha's answer. In eg the single/double slit experiment the observation is the same and opening/closing a second slit "at a distance" alters the observed result - even when a single eg photon is used and the information transfer occurs faster than C. The photon "KNOWS" what is happening at distances far enough away that no information can 'tell it' so in classical terms. || Your answer is essentially saying that the observers behaviour changes when eg a slit is opened or closed and the setup-change causes the observer to change behaviour. This MAY be true :-) !
    $endgroup$
    – Russell McMahon
    Apr 21 at 2:49


















3












$begingroup$

Light is assumed to be particle or wave to describe the observed phenomenon. Light is what it is and doesn't choose. However, one can consider light to be a particle when the interaction is with matter & as a wave when it interacts with itself.






share|cite|improve this answer








New contributor




navnth is a new contributor to this site. Take care in asking for clarification, commenting, and answering.
Check out our Code of Conduct.






$endgroup$




















    2












    $begingroup$

    It doesn't is the short answer, as it is really neither. Basically, waves and particles are things you see on the classical scale, they model behaviour on macroscopic scales.



    Objects don't behave in a classical way any more when we shrink down to very small scales and they obey very different dynamics which seem strange to us because we do not live on that scale and are only able to access things on that scale in quite a dim way via theory and experiment. It's kind of common sense when you think about it, just stop expecting that things will behave in a classical way when you going down to those length scales.



    Also forgot the wave-particle duality, as it does not even make sense. It's like a blind person touching an elephant on the trunk and then touching it again on the leg and saying that the elephant follows trunk-leg duality.






    share|cite|improve this answer









    $endgroup$




















      2












      $begingroup$

      It doesn’t.



      Light, as you may know, is made up of photons, and photons are particles. At the same time, all particles are waves. Yes, that's right—particles are not points or spheres or anything else really solid moving around, but waves, which interact with each other in various ways.



      Now, these waves may sometimes behave indistinguishably from points moving around in space, and may therefore be approximated as such—what we like to refer to as point particles—but they are fundamentally still always waves.



      So, a photon is always both a wave and a particle; these are not mutually exclusive (in fact, as I have explained now, particles are a subset of waves). This is what is known as the wave–particle duality.






      share|cite|improve this answer











      $endgroup$




















        1












        $begingroup$

        binaryfunt and Ben Crowell's answer is closest to the truth. Let me add a few things.



        You get confused because you read sentences like, light behaves as a wave when it travels in space and as a particle when it interferes with matter. That is not true. In reality light has characteristics of both waves and particles at the same time. The truth is, we do not know what actually light is, or what it is really made of.



        In reality, light is not what decides or behaves different ways, it is us, the ones preparing the experiments, who decide whether we will select from the observed (both wave or particle like) nature the more interesting wave or particle like characteristics, and analyze those for certain purposes.



        What we try to do, is model with our mathematical ways the real phenomenon in our physical world, light itself.



        The reason you get confused is because there are two ways to really model this phenomenon:



        1. classical EM waves


        2. QM particles, photons


        You would think they do not match, but that is not true. In reality the two methods come together perfectly, as we build up the classical EM waves from herds of QM photons.



        In our currently accepted theory, the Standard Model, we talk about elementary particles, that have no internal structure, nor spatial extent, and we call them point like particles. One of these particles is the photon. We call it the quanta of the EM wave.



        It is very important to see that the basic confusion lies in whether you want to analyze the wave or the particle characteristics of light in a certain experiment.



        We usually model light in two ways, and that is what confuses you, these to ways are whether it travels or when it interacts with matter. This is basically a misconception, because light could be analyzed to show wave and particles characteristics both when it travels and when it interacts with matter.



        It comes from a misconception to choose between travel and interaction, but for the sake of argument, lets see:



        1. travel

        You can use Maxwell's original equations or use QFT to treat photons and light as excitation of the EM field, that travels through space, and this propagation is modeled in our theories as a wave. This is because this wave model is what best fits the experimental data. Now we can show that the QM or QFT field excitation model of the photon propagating through space is the same fitting the data perfectly from the experiments.



        So you see light has both wave and particle characteristics even as it travels through space.



        1. interaction with matter

        Now when a photon interacts with an atom, three things can happen:



        1. elastic scattering, the photon gives all its energy to the atom and changes angle


        2. inelastic scattering, the photon gives part of its energy to the atom and changes angle


        3. absorption, the photon gives all its energy to the absorbing atom/electron


        For all three, you can see that light both shows wave and particle characteristics. It is a misconception that the experiments about interaction are about the particle characteristics alone. Even in these three cases, when light interacts with matter, it behaves as a wave, in certain cases it changes angle, goes through slits, splits into partial waves, these partial waves interact with themselves, create interference patterns. Even in the case of absorption, when the photon ceises to exist, and transforms into the kinetic energy of the electron, it shows wave nature and particle both since the absorbing atomic system and electron both have wave and particle characteristics too.



        So there is no reason to really make a hard decision whether light would choose, it does not. It has both wave and particle characteristics as intrinsic, and it is up to us who prepare the experiment and analyze the data which we want to focus on.






        share|cite|improve this answer









        $endgroup$











          protected by ACuriousMind Apr 19 at 18:49



          Thank you for your interest in this question.
          Because it has attracted low-quality or spam answers that had to be removed, posting an answer now requires 10 reputation on this site (the association bonus does not count).



          Would you like to answer one of these unanswered questions instead?














          11 Answers
          11






          active

          oldest

          votes








          11 Answers
          11






          active

          oldest

          votes









          active

          oldest

          votes






          active

          oldest

          votes









          134












          $begingroup$

          In fact, light is not really a wave or a particle. It is what it is; it's this strange thing that we model as a wave or a particle in order to make sense of its behaviour, depending on the scenario of interest.



          At the end of the day, it's the same story with all theories in physics. Planets don't "choose" to follow Newtonian mechanics or general relativity. Instead, we can model their motion as Newtonian if we want to calculate something like where Mars will be in 2 weeks, but need to use general relativity if we want to explain why the atomic clock on a satellite runs slow compared to one on the ground.



          Light doesn't "choose" to be a wave or a particle. Instead, we model it as a wave when we want to explain (or calculate) interference, but need to model it as a particle when we want to explain (or calculate) the photoelectric effect.






          share|cite|improve this answer











          $endgroup$












          • $begingroup$
            Comments are not for extended discussion; this conversation has been moved to chat.
            $endgroup$
            – ACuriousMind
            Apr 22 at 10:09















          134












          $begingroup$

          In fact, light is not really a wave or a particle. It is what it is; it's this strange thing that we model as a wave or a particle in order to make sense of its behaviour, depending on the scenario of interest.



          At the end of the day, it's the same story with all theories in physics. Planets don't "choose" to follow Newtonian mechanics or general relativity. Instead, we can model their motion as Newtonian if we want to calculate something like where Mars will be in 2 weeks, but need to use general relativity if we want to explain why the atomic clock on a satellite runs slow compared to one on the ground.



          Light doesn't "choose" to be a wave or a particle. Instead, we model it as a wave when we want to explain (or calculate) interference, but need to model it as a particle when we want to explain (or calculate) the photoelectric effect.






          share|cite|improve this answer











          $endgroup$












          • $begingroup$
            Comments are not for extended discussion; this conversation has been moved to chat.
            $endgroup$
            – ACuriousMind
            Apr 22 at 10:09













          134












          134








          134





          $begingroup$

          In fact, light is not really a wave or a particle. It is what it is; it's this strange thing that we model as a wave or a particle in order to make sense of its behaviour, depending on the scenario of interest.



          At the end of the day, it's the same story with all theories in physics. Planets don't "choose" to follow Newtonian mechanics or general relativity. Instead, we can model their motion as Newtonian if we want to calculate something like where Mars will be in 2 weeks, but need to use general relativity if we want to explain why the atomic clock on a satellite runs slow compared to one on the ground.



          Light doesn't "choose" to be a wave or a particle. Instead, we model it as a wave when we want to explain (or calculate) interference, but need to model it as a particle when we want to explain (or calculate) the photoelectric effect.






          share|cite|improve this answer











          $endgroup$



          In fact, light is not really a wave or a particle. It is what it is; it's this strange thing that we model as a wave or a particle in order to make sense of its behaviour, depending on the scenario of interest.



          At the end of the day, it's the same story with all theories in physics. Planets don't "choose" to follow Newtonian mechanics or general relativity. Instead, we can model their motion as Newtonian if we want to calculate something like where Mars will be in 2 weeks, but need to use general relativity if we want to explain why the atomic clock on a satellite runs slow compared to one on the ground.



          Light doesn't "choose" to be a wave or a particle. Instead, we model it as a wave when we want to explain (or calculate) interference, but need to model it as a particle when we want to explain (or calculate) the photoelectric effect.







          share|cite|improve this answer














          share|cite|improve this answer



          share|cite|improve this answer








          edited Apr 18 at 19:26

























          answered Apr 18 at 19:16









          binaryfuntbinaryfunt

          1,1752922




          1,1752922











          • $begingroup$
            Comments are not for extended discussion; this conversation has been moved to chat.
            $endgroup$
            – ACuriousMind
            Apr 22 at 10:09
















          • $begingroup$
            Comments are not for extended discussion; this conversation has been moved to chat.
            $endgroup$
            – ACuriousMind
            Apr 22 at 10:09















          $begingroup$
          Comments are not for extended discussion; this conversation has been moved to chat.
          $endgroup$
          – ACuriousMind
          Apr 22 at 10:09




          $begingroup$
          Comments are not for extended discussion; this conversation has been moved to chat.
          $endgroup$
          – ACuriousMind
          Apr 22 at 10:09











          17












          $begingroup$


          How does light 'choose' where to be a wave and where to be a particle?




          It doesn't, YOU do. That's really the entire "weirdness" of quantum right there.



          It's not entirely crazy. If you measure a car with a scale it will tell you it's 1200 kg, and if you measure it with a spectrometer it will tell you its red. This is perfectly natural.



          The thing that makes quantum weird is that you can measure the same thing twice and get two different answers. More weird, some of those measurements are linked to each other so if you measure one the other changes.



          Its as if you measured the weight of your car and the length changed. And then you measured the length and the weight changed. This precise thing happens in quantum, for instance, the position and momentum of a particle are linked in this way



          In any event, the wave-or-particle nature is entirely up to you. Which nature you see is based on the experiment you use, not the photon itself.



          The idea that something you do has this "radical" effect on the outcome is what drives everyone mad in QM.






          share|cite|improve this answer











          $endgroup$








          • 1




            $begingroup$
            This could be a nice answer if you could elaborate a bit.
            $endgroup$
            – Gert
            Apr 18 at 15:11










          • $begingroup$
            If you can choose the state of light (for eg), would that mean that theoretically we could influence light to behave as a wave and not as photons in phenomenon such as the photoelectric effect?
            $endgroup$
            – d_g
            Apr 18 at 15:15






          • 12




            $begingroup$
            Ahhh but there you go... how do you measure the photoelectric effect? By measuring its energy. Presto, it looks like a particle. Take that exact same photon and put it in a double-slit? Presto, looks like a wave. But these terms, "particle" and "wave" are the real problem, you're using classical terms that simply don't describe what's really happening. You would have a similar difficulty trying to describe a car to someone who only saw horses and tried to say the wheels were sort of like legs... how often do you need new horseshoes?
            $endgroup$
            – Maury Markowitz
            Apr 18 at 15:18






          • 1




            $begingroup$
            If you measure the weight of car with a beam scale first and then with a spring scale, you'll get the same weight twice but in the second case your car may stretch itself under its own weight. So even in "macro" the way of measuring can have an impact on the properties of the object observed.
            $endgroup$
            – Echox
            Apr 18 at 20:33






          • 2




            $begingroup$
            @MauryMarkowitz you need new horseshoes twice per year, once to make your horse not slip on ice, once to make him eat less while running on not-ice.
            $endgroup$
            – John Dvorak
            Apr 19 at 12:48















          17












          $begingroup$


          How does light 'choose' where to be a wave and where to be a particle?




          It doesn't, YOU do. That's really the entire "weirdness" of quantum right there.



          It's not entirely crazy. If you measure a car with a scale it will tell you it's 1200 kg, and if you measure it with a spectrometer it will tell you its red. This is perfectly natural.



          The thing that makes quantum weird is that you can measure the same thing twice and get two different answers. More weird, some of those measurements are linked to each other so if you measure one the other changes.



          Its as if you measured the weight of your car and the length changed. And then you measured the length and the weight changed. This precise thing happens in quantum, for instance, the position and momentum of a particle are linked in this way



          In any event, the wave-or-particle nature is entirely up to you. Which nature you see is based on the experiment you use, not the photon itself.



          The idea that something you do has this "radical" effect on the outcome is what drives everyone mad in QM.






          share|cite|improve this answer











          $endgroup$








          • 1




            $begingroup$
            This could be a nice answer if you could elaborate a bit.
            $endgroup$
            – Gert
            Apr 18 at 15:11










          • $begingroup$
            If you can choose the state of light (for eg), would that mean that theoretically we could influence light to behave as a wave and not as photons in phenomenon such as the photoelectric effect?
            $endgroup$
            – d_g
            Apr 18 at 15:15






          • 12




            $begingroup$
            Ahhh but there you go... how do you measure the photoelectric effect? By measuring its energy. Presto, it looks like a particle. Take that exact same photon and put it in a double-slit? Presto, looks like a wave. But these terms, "particle" and "wave" are the real problem, you're using classical terms that simply don't describe what's really happening. You would have a similar difficulty trying to describe a car to someone who only saw horses and tried to say the wheels were sort of like legs... how often do you need new horseshoes?
            $endgroup$
            – Maury Markowitz
            Apr 18 at 15:18






          • 1




            $begingroup$
            If you measure the weight of car with a beam scale first and then with a spring scale, you'll get the same weight twice but in the second case your car may stretch itself under its own weight. So even in "macro" the way of measuring can have an impact on the properties of the object observed.
            $endgroup$
            – Echox
            Apr 18 at 20:33






          • 2




            $begingroup$
            @MauryMarkowitz you need new horseshoes twice per year, once to make your horse not slip on ice, once to make him eat less while running on not-ice.
            $endgroup$
            – John Dvorak
            Apr 19 at 12:48













          17












          17








          17





          $begingroup$


          How does light 'choose' where to be a wave and where to be a particle?




          It doesn't, YOU do. That's really the entire "weirdness" of quantum right there.



          It's not entirely crazy. If you measure a car with a scale it will tell you it's 1200 kg, and if you measure it with a spectrometer it will tell you its red. This is perfectly natural.



          The thing that makes quantum weird is that you can measure the same thing twice and get two different answers. More weird, some of those measurements are linked to each other so if you measure one the other changes.



          Its as if you measured the weight of your car and the length changed. And then you measured the length and the weight changed. This precise thing happens in quantum, for instance, the position and momentum of a particle are linked in this way



          In any event, the wave-or-particle nature is entirely up to you. Which nature you see is based on the experiment you use, not the photon itself.



          The idea that something you do has this "radical" effect on the outcome is what drives everyone mad in QM.






          share|cite|improve this answer











          $endgroup$




          How does light 'choose' where to be a wave and where to be a particle?




          It doesn't, YOU do. That's really the entire "weirdness" of quantum right there.



          It's not entirely crazy. If you measure a car with a scale it will tell you it's 1200 kg, and if you measure it with a spectrometer it will tell you its red. This is perfectly natural.



          The thing that makes quantum weird is that you can measure the same thing twice and get two different answers. More weird, some of those measurements are linked to each other so if you measure one the other changes.



          Its as if you measured the weight of your car and the length changed. And then you measured the length and the weight changed. This precise thing happens in quantum, for instance, the position and momentum of a particle are linked in this way



          In any event, the wave-or-particle nature is entirely up to you. Which nature you see is based on the experiment you use, not the photon itself.



          The idea that something you do has this "radical" effect on the outcome is what drives everyone mad in QM.







          share|cite|improve this answer














          share|cite|improve this answer



          share|cite|improve this answer








          edited Apr 18 at 15:16

























          answered Apr 18 at 15:07









          Maury MarkowitzMaury Markowitz

          4,5031628




          4,5031628







          • 1




            $begingroup$
            This could be a nice answer if you could elaborate a bit.
            $endgroup$
            – Gert
            Apr 18 at 15:11










          • $begingroup$
            If you can choose the state of light (for eg), would that mean that theoretically we could influence light to behave as a wave and not as photons in phenomenon such as the photoelectric effect?
            $endgroup$
            – d_g
            Apr 18 at 15:15






          • 12




            $begingroup$
            Ahhh but there you go... how do you measure the photoelectric effect? By measuring its energy. Presto, it looks like a particle. Take that exact same photon and put it in a double-slit? Presto, looks like a wave. But these terms, "particle" and "wave" are the real problem, you're using classical terms that simply don't describe what's really happening. You would have a similar difficulty trying to describe a car to someone who only saw horses and tried to say the wheels were sort of like legs... how often do you need new horseshoes?
            $endgroup$
            – Maury Markowitz
            Apr 18 at 15:18






          • 1




            $begingroup$
            If you measure the weight of car with a beam scale first and then with a spring scale, you'll get the same weight twice but in the second case your car may stretch itself under its own weight. So even in "macro" the way of measuring can have an impact on the properties of the object observed.
            $endgroup$
            – Echox
            Apr 18 at 20:33






          • 2




            $begingroup$
            @MauryMarkowitz you need new horseshoes twice per year, once to make your horse not slip on ice, once to make him eat less while running on not-ice.
            $endgroup$
            – John Dvorak
            Apr 19 at 12:48












          • 1




            $begingroup$
            This could be a nice answer if you could elaborate a bit.
            $endgroup$
            – Gert
            Apr 18 at 15:11










          • $begingroup$
            If you can choose the state of light (for eg), would that mean that theoretically we could influence light to behave as a wave and not as photons in phenomenon such as the photoelectric effect?
            $endgroup$
            – d_g
            Apr 18 at 15:15






          • 12




            $begingroup$
            Ahhh but there you go... how do you measure the photoelectric effect? By measuring its energy. Presto, it looks like a particle. Take that exact same photon and put it in a double-slit? Presto, looks like a wave. But these terms, "particle" and "wave" are the real problem, you're using classical terms that simply don't describe what's really happening. You would have a similar difficulty trying to describe a car to someone who only saw horses and tried to say the wheels were sort of like legs... how often do you need new horseshoes?
            $endgroup$
            – Maury Markowitz
            Apr 18 at 15:18






          • 1




            $begingroup$
            If you measure the weight of car with a beam scale first and then with a spring scale, you'll get the same weight twice but in the second case your car may stretch itself under its own weight. So even in "macro" the way of measuring can have an impact on the properties of the object observed.
            $endgroup$
            – Echox
            Apr 18 at 20:33






          • 2




            $begingroup$
            @MauryMarkowitz you need new horseshoes twice per year, once to make your horse not slip on ice, once to make him eat less while running on not-ice.
            $endgroup$
            – John Dvorak
            Apr 19 at 12:48







          1




          1




          $begingroup$
          This could be a nice answer if you could elaborate a bit.
          $endgroup$
          – Gert
          Apr 18 at 15:11




          $begingroup$
          This could be a nice answer if you could elaborate a bit.
          $endgroup$
          – Gert
          Apr 18 at 15:11












          $begingroup$
          If you can choose the state of light (for eg), would that mean that theoretically we could influence light to behave as a wave and not as photons in phenomenon such as the photoelectric effect?
          $endgroup$
          – d_g
          Apr 18 at 15:15




          $begingroup$
          If you can choose the state of light (for eg), would that mean that theoretically we could influence light to behave as a wave and not as photons in phenomenon such as the photoelectric effect?
          $endgroup$
          – d_g
          Apr 18 at 15:15




          12




          12




          $begingroup$
          Ahhh but there you go... how do you measure the photoelectric effect? By measuring its energy. Presto, it looks like a particle. Take that exact same photon and put it in a double-slit? Presto, looks like a wave. But these terms, "particle" and "wave" are the real problem, you're using classical terms that simply don't describe what's really happening. You would have a similar difficulty trying to describe a car to someone who only saw horses and tried to say the wheels were sort of like legs... how often do you need new horseshoes?
          $endgroup$
          – Maury Markowitz
          Apr 18 at 15:18




          $begingroup$
          Ahhh but there you go... how do you measure the photoelectric effect? By measuring its energy. Presto, it looks like a particle. Take that exact same photon and put it in a double-slit? Presto, looks like a wave. But these terms, "particle" and "wave" are the real problem, you're using classical terms that simply don't describe what's really happening. You would have a similar difficulty trying to describe a car to someone who only saw horses and tried to say the wheels were sort of like legs... how often do you need new horseshoes?
          $endgroup$
          – Maury Markowitz
          Apr 18 at 15:18




          1




          1




          $begingroup$
          If you measure the weight of car with a beam scale first and then with a spring scale, you'll get the same weight twice but in the second case your car may stretch itself under its own weight. So even in "macro" the way of measuring can have an impact on the properties of the object observed.
          $endgroup$
          – Echox
          Apr 18 at 20:33




          $begingroup$
          If you measure the weight of car with a beam scale first and then with a spring scale, you'll get the same weight twice but in the second case your car may stretch itself under its own weight. So even in "macro" the way of measuring can have an impact on the properties of the object observed.
          $endgroup$
          – Echox
          Apr 18 at 20:33




          2




          2




          $begingroup$
          @MauryMarkowitz you need new horseshoes twice per year, once to make your horse not slip on ice, once to make him eat less while running on not-ice.
          $endgroup$
          – John Dvorak
          Apr 19 at 12:48




          $begingroup$
          @MauryMarkowitz you need new horseshoes twice per year, once to make your horse not slip on ice, once to make him eat less while running on not-ice.
          $endgroup$
          – John Dvorak
          Apr 19 at 12:48











          12












          $begingroup$

          Light always behaves as a wave. Particles can be thought of as a combination of waves, a wave packet. What determines which behavior you'll get is the length scale of the system the light is interacting with and the wavelength of the impinging light.



          Suppose you have a series of Bowling Balls suspended in the air making a wall, but with spaces between. Shoot a bunch of 2mm diameter ball bearings (BBs) at the bowling balls. Some will pass through the wall, hitting no bowling balls. Some will hit a ball and bounce in some direction.



          Pay close attention to where the BBs come from and their initial speed and where they end up, and you will be able to tell the shape, size, and position of the bowling balls.



          Reverse the problem. Have a bunch of suspended BBs in the air and shoot bowling balls at them. Each bowling ball will hit multiple BBs. The outgoing trajectory won't tell you much about the BBs.



          Strike a tuning fork, it vibrates, making a characteristic sound. Play that note at high volume, you can set the tuning fork to vibrating. It doesn't start to vibrate at just any wavelength.



          To a particle, you can associate the De Broglie wavelength, $h/p$, where $p$ is the momentum. The higher the momentum, the lower the wavelength, the more particle like. Whereas macroscopic but small openings can be used with light in the double slit experiment, you need electron crystallography to demonstrate the same effects with electrons: Electron diffraction



          If you have a large wavelength of light, it will interact with multiple particles of a system depending on that system. If the wavelength is sufficiently small, and so the energy sufficiently high, it can interact with a single electron instead of multiple electrons, imparting all its energy to an electron and resulting in the photoelectric effect, a particle-particle scattering effect. Change the wave length and the interaction takes on a more classical form.



          In addition to the wavelengths involved you want to pay attention to the number of photons available. The fewer the photons interacting with the system, the more quantum-like it is. The higher the density of photons, the more classical the light will behave. For a more detailed explanation on the barrier between quantum vs. classical behavior, see the intro and first chapter of Griffith's text on electromagnetism.



          In short, the behavior you get depends on the De Broglie wavelength of the light/particle, how many particles are inbound and how the lengths scales of the target compare with the De Broglie wavelength.






          share|cite|improve this answer











          $endgroup$












          • $begingroup$
            Any mathematical account of the correspondence principle will also give insight into how classical effects emerge in a limit from quantum ones.
            $endgroup$
            – J.G.
            Apr 18 at 19:14






          • 2




            $begingroup$
            @R. Romero did you mean baseballs or ball bearings when you said "BBs" in your original answer?
            $endgroup$
            – binaryfunt
            Apr 18 at 20:13










          • $begingroup$
            Ball bearings. sorry, I should have been clear. Tiny ball bearings a few cm in diameter.
            $endgroup$
            – R. Romero
            Apr 18 at 20:20






          • 1




            $begingroup$
            -1 this answer is completely wrong, I don't understand why it gets so many upvotes: "Particles can be thought of as a combination of waves, a wave packet" is wrong. A wave packet is a completely different thing; moreover there are no photons in QM (they emerge in QFT only). Last but not the least, the de Broglie wavelength has nothing to do with the "wave nature". I don't want to sound rude but this answer is a salad of physics misconceptions and should be deeply amended.
            $endgroup$
            – gented
            Apr 21 at 11:27







          • 1




            $begingroup$
            Einstein's paper doesn't demonstrate quanta and Bohr's models aren't the way quantum physics is formulated: they were initial ad hoc models to make things work in a certain way. Quantum mechanics and quantum field theory have been formalised since then and all those quantities (photons, quanta and all the rest) have a precise meaning which conceptually isn't what you have described.
            $endgroup$
            – gented
            2 days ago
















          12












          $begingroup$

          Light always behaves as a wave. Particles can be thought of as a combination of waves, a wave packet. What determines which behavior you'll get is the length scale of the system the light is interacting with and the wavelength of the impinging light.



          Suppose you have a series of Bowling Balls suspended in the air making a wall, but with spaces between. Shoot a bunch of 2mm diameter ball bearings (BBs) at the bowling balls. Some will pass through the wall, hitting no bowling balls. Some will hit a ball and bounce in some direction.



          Pay close attention to where the BBs come from and their initial speed and where they end up, and you will be able to tell the shape, size, and position of the bowling balls.



          Reverse the problem. Have a bunch of suspended BBs in the air and shoot bowling balls at them. Each bowling ball will hit multiple BBs. The outgoing trajectory won't tell you much about the BBs.



          Strike a tuning fork, it vibrates, making a characteristic sound. Play that note at high volume, you can set the tuning fork to vibrating. It doesn't start to vibrate at just any wavelength.



          To a particle, you can associate the De Broglie wavelength, $h/p$, where $p$ is the momentum. The higher the momentum, the lower the wavelength, the more particle like. Whereas macroscopic but small openings can be used with light in the double slit experiment, you need electron crystallography to demonstrate the same effects with electrons: Electron diffraction



          If you have a large wavelength of light, it will interact with multiple particles of a system depending on that system. If the wavelength is sufficiently small, and so the energy sufficiently high, it can interact with a single electron instead of multiple electrons, imparting all its energy to an electron and resulting in the photoelectric effect, a particle-particle scattering effect. Change the wave length and the interaction takes on a more classical form.



          In addition to the wavelengths involved you want to pay attention to the number of photons available. The fewer the photons interacting with the system, the more quantum-like it is. The higher the density of photons, the more classical the light will behave. For a more detailed explanation on the barrier between quantum vs. classical behavior, see the intro and first chapter of Griffith's text on electromagnetism.



          In short, the behavior you get depends on the De Broglie wavelength of the light/particle, how many particles are inbound and how the lengths scales of the target compare with the De Broglie wavelength.






          share|cite|improve this answer











          $endgroup$












          • $begingroup$
            Any mathematical account of the correspondence principle will also give insight into how classical effects emerge in a limit from quantum ones.
            $endgroup$
            – J.G.
            Apr 18 at 19:14






          • 2




            $begingroup$
            @R. Romero did you mean baseballs or ball bearings when you said "BBs" in your original answer?
            $endgroup$
            – binaryfunt
            Apr 18 at 20:13










          • $begingroup$
            Ball bearings. sorry, I should have been clear. Tiny ball bearings a few cm in diameter.
            $endgroup$
            – R. Romero
            Apr 18 at 20:20






          • 1




            $begingroup$
            -1 this answer is completely wrong, I don't understand why it gets so many upvotes: "Particles can be thought of as a combination of waves, a wave packet" is wrong. A wave packet is a completely different thing; moreover there are no photons in QM (they emerge in QFT only). Last but not the least, the de Broglie wavelength has nothing to do with the "wave nature". I don't want to sound rude but this answer is a salad of physics misconceptions and should be deeply amended.
            $endgroup$
            – gented
            Apr 21 at 11:27







          • 1




            $begingroup$
            Einstein's paper doesn't demonstrate quanta and Bohr's models aren't the way quantum physics is formulated: they were initial ad hoc models to make things work in a certain way. Quantum mechanics and quantum field theory have been formalised since then and all those quantities (photons, quanta and all the rest) have a precise meaning which conceptually isn't what you have described.
            $endgroup$
            – gented
            2 days ago














          12












          12








          12





          $begingroup$

          Light always behaves as a wave. Particles can be thought of as a combination of waves, a wave packet. What determines which behavior you'll get is the length scale of the system the light is interacting with and the wavelength of the impinging light.



          Suppose you have a series of Bowling Balls suspended in the air making a wall, but with spaces between. Shoot a bunch of 2mm diameter ball bearings (BBs) at the bowling balls. Some will pass through the wall, hitting no bowling balls. Some will hit a ball and bounce in some direction.



          Pay close attention to where the BBs come from and their initial speed and where they end up, and you will be able to tell the shape, size, and position of the bowling balls.



          Reverse the problem. Have a bunch of suspended BBs in the air and shoot bowling balls at them. Each bowling ball will hit multiple BBs. The outgoing trajectory won't tell you much about the BBs.



          Strike a tuning fork, it vibrates, making a characteristic sound. Play that note at high volume, you can set the tuning fork to vibrating. It doesn't start to vibrate at just any wavelength.



          To a particle, you can associate the De Broglie wavelength, $h/p$, where $p$ is the momentum. The higher the momentum, the lower the wavelength, the more particle like. Whereas macroscopic but small openings can be used with light in the double slit experiment, you need electron crystallography to demonstrate the same effects with electrons: Electron diffraction



          If you have a large wavelength of light, it will interact with multiple particles of a system depending on that system. If the wavelength is sufficiently small, and so the energy sufficiently high, it can interact with a single electron instead of multiple electrons, imparting all its energy to an electron and resulting in the photoelectric effect, a particle-particle scattering effect. Change the wave length and the interaction takes on a more classical form.



          In addition to the wavelengths involved you want to pay attention to the number of photons available. The fewer the photons interacting with the system, the more quantum-like it is. The higher the density of photons, the more classical the light will behave. For a more detailed explanation on the barrier between quantum vs. classical behavior, see the intro and first chapter of Griffith's text on electromagnetism.



          In short, the behavior you get depends on the De Broglie wavelength of the light/particle, how many particles are inbound and how the lengths scales of the target compare with the De Broglie wavelength.






          share|cite|improve this answer











          $endgroup$



          Light always behaves as a wave. Particles can be thought of as a combination of waves, a wave packet. What determines which behavior you'll get is the length scale of the system the light is interacting with and the wavelength of the impinging light.



          Suppose you have a series of Bowling Balls suspended in the air making a wall, but with spaces between. Shoot a bunch of 2mm diameter ball bearings (BBs) at the bowling balls. Some will pass through the wall, hitting no bowling balls. Some will hit a ball and bounce in some direction.



          Pay close attention to where the BBs come from and their initial speed and where they end up, and you will be able to tell the shape, size, and position of the bowling balls.



          Reverse the problem. Have a bunch of suspended BBs in the air and shoot bowling balls at them. Each bowling ball will hit multiple BBs. The outgoing trajectory won't tell you much about the BBs.



          Strike a tuning fork, it vibrates, making a characteristic sound. Play that note at high volume, you can set the tuning fork to vibrating. It doesn't start to vibrate at just any wavelength.



          To a particle, you can associate the De Broglie wavelength, $h/p$, where $p$ is the momentum. The higher the momentum, the lower the wavelength, the more particle like. Whereas macroscopic but small openings can be used with light in the double slit experiment, you need electron crystallography to demonstrate the same effects with electrons: Electron diffraction



          If you have a large wavelength of light, it will interact with multiple particles of a system depending on that system. If the wavelength is sufficiently small, and so the energy sufficiently high, it can interact with a single electron instead of multiple electrons, imparting all its energy to an electron and resulting in the photoelectric effect, a particle-particle scattering effect. Change the wave length and the interaction takes on a more classical form.



          In addition to the wavelengths involved you want to pay attention to the number of photons available. The fewer the photons interacting with the system, the more quantum-like it is. The higher the density of photons, the more classical the light will behave. For a more detailed explanation on the barrier between quantum vs. classical behavior, see the intro and first chapter of Griffith's text on electromagnetism.



          In short, the behavior you get depends on the De Broglie wavelength of the light/particle, how many particles are inbound and how the lengths scales of the target compare with the De Broglie wavelength.







          share|cite|improve this answer














          share|cite|improve this answer



          share|cite|improve this answer








          edited Apr 18 at 20:33

























          answered Apr 18 at 15:22









          R. RomeroR. Romero

          66110




          66110











          • $begingroup$
            Any mathematical account of the correspondence principle will also give insight into how classical effects emerge in a limit from quantum ones.
            $endgroup$
            – J.G.
            Apr 18 at 19:14






          • 2




            $begingroup$
            @R. Romero did you mean baseballs or ball bearings when you said "BBs" in your original answer?
            $endgroup$
            – binaryfunt
            Apr 18 at 20:13










          • $begingroup$
            Ball bearings. sorry, I should have been clear. Tiny ball bearings a few cm in diameter.
            $endgroup$
            – R. Romero
            Apr 18 at 20:20






          • 1




            $begingroup$
            -1 this answer is completely wrong, I don't understand why it gets so many upvotes: "Particles can be thought of as a combination of waves, a wave packet" is wrong. A wave packet is a completely different thing; moreover there are no photons in QM (they emerge in QFT only). Last but not the least, the de Broglie wavelength has nothing to do with the "wave nature". I don't want to sound rude but this answer is a salad of physics misconceptions and should be deeply amended.
            $endgroup$
            – gented
            Apr 21 at 11:27







          • 1




            $begingroup$
            Einstein's paper doesn't demonstrate quanta and Bohr's models aren't the way quantum physics is formulated: they were initial ad hoc models to make things work in a certain way. Quantum mechanics and quantum field theory have been formalised since then and all those quantities (photons, quanta and all the rest) have a precise meaning which conceptually isn't what you have described.
            $endgroup$
            – gented
            2 days ago

















          • $begingroup$
            Any mathematical account of the correspondence principle will also give insight into how classical effects emerge in a limit from quantum ones.
            $endgroup$
            – J.G.
            Apr 18 at 19:14






          • 2




            $begingroup$
            @R. Romero did you mean baseballs or ball bearings when you said "BBs" in your original answer?
            $endgroup$
            – binaryfunt
            Apr 18 at 20:13










          • $begingroup$
            Ball bearings. sorry, I should have been clear. Tiny ball bearings a few cm in diameter.
            $endgroup$
            – R. Romero
            Apr 18 at 20:20






          • 1




            $begingroup$
            -1 this answer is completely wrong, I don't understand why it gets so many upvotes: "Particles can be thought of as a combination of waves, a wave packet" is wrong. A wave packet is a completely different thing; moreover there are no photons in QM (they emerge in QFT only). Last but not the least, the de Broglie wavelength has nothing to do with the "wave nature". I don't want to sound rude but this answer is a salad of physics misconceptions and should be deeply amended.
            $endgroup$
            – gented
            Apr 21 at 11:27







          • 1




            $begingroup$
            Einstein's paper doesn't demonstrate quanta and Bohr's models aren't the way quantum physics is formulated: they were initial ad hoc models to make things work in a certain way. Quantum mechanics and quantum field theory have been formalised since then and all those quantities (photons, quanta and all the rest) have a precise meaning which conceptually isn't what you have described.
            $endgroup$
            – gented
            2 days ago
















          $begingroup$
          Any mathematical account of the correspondence principle will also give insight into how classical effects emerge in a limit from quantum ones.
          $endgroup$
          – J.G.
          Apr 18 at 19:14




          $begingroup$
          Any mathematical account of the correspondence principle will also give insight into how classical effects emerge in a limit from quantum ones.
          $endgroup$
          – J.G.
          Apr 18 at 19:14




          2




          2




          $begingroup$
          @R. Romero did you mean baseballs or ball bearings when you said "BBs" in your original answer?
          $endgroup$
          – binaryfunt
          Apr 18 at 20:13




          $begingroup$
          @R. Romero did you mean baseballs or ball bearings when you said "BBs" in your original answer?
          $endgroup$
          – binaryfunt
          Apr 18 at 20:13












          $begingroup$
          Ball bearings. sorry, I should have been clear. Tiny ball bearings a few cm in diameter.
          $endgroup$
          – R. Romero
          Apr 18 at 20:20




          $begingroup$
          Ball bearings. sorry, I should have been clear. Tiny ball bearings a few cm in diameter.
          $endgroup$
          – R. Romero
          Apr 18 at 20:20




          1




          1




          $begingroup$
          -1 this answer is completely wrong, I don't understand why it gets so many upvotes: "Particles can be thought of as a combination of waves, a wave packet" is wrong. A wave packet is a completely different thing; moreover there are no photons in QM (they emerge in QFT only). Last but not the least, the de Broglie wavelength has nothing to do with the "wave nature". I don't want to sound rude but this answer is a salad of physics misconceptions and should be deeply amended.
          $endgroup$
          – gented
          Apr 21 at 11:27





          $begingroup$
          -1 this answer is completely wrong, I don't understand why it gets so many upvotes: "Particles can be thought of as a combination of waves, a wave packet" is wrong. A wave packet is a completely different thing; moreover there are no photons in QM (they emerge in QFT only). Last but not the least, the de Broglie wavelength has nothing to do with the "wave nature". I don't want to sound rude but this answer is a salad of physics misconceptions and should be deeply amended.
          $endgroup$
          – gented
          Apr 21 at 11:27





          1




          1




          $begingroup$
          Einstein's paper doesn't demonstrate quanta and Bohr's models aren't the way quantum physics is formulated: they were initial ad hoc models to make things work in a certain way. Quantum mechanics and quantum field theory have been formalised since then and all those quantities (photons, quanta and all the rest) have a precise meaning which conceptually isn't what you have described.
          $endgroup$
          – gented
          2 days ago





          $begingroup$
          Einstein's paper doesn't demonstrate quanta and Bohr's models aren't the way quantum physics is formulated: they were initial ad hoc models to make things work in a certain way. Quantum mechanics and quantum field theory have been formalised since then and all those quantities (photons, quanta and all the rest) have a precise meaning which conceptually isn't what you have described.
          $endgroup$
          – gented
          2 days ago












          8












          $begingroup$


          How does light 'choose' where to be a wave and where to be a particle?




          It doesn't. It always behaves as a wave (obeying the principle of superposition), and it always behaves as a particle (particle number being quantized).



          It sounds like you may have been influenced by someone who told you that light behaves like a particle in some experiments, and like a wave in others. That's false.






          share|cite|improve this answer









          $endgroup$








          • 4




            $begingroup$
            How do you address the fact that the photoelectric effect doesn't make sense if you consider light as a wave though? I don't think it is fine to say it always behaves like a wave, unless you mean to say the only consistent wave behavior is superposition
            $endgroup$
            – Aaron Stevens
            Apr 18 at 15:40










          • $begingroup$
            @AaronStevens The photoelectric effect doesn't "disprove" a wave description. If I place a guitar on a table and dust settles on one string, which I then pluck, whether I displace no specks of dust, one of them or several depends on how hard I pluck. But what caused any such displacement? A wave transmitting energy down the string. It's just that in QM, this is also explicable in terms of individual photons' absorption. This is why I recommend learning QFT, which makes better sense of how the wave/particle aspects mesh together.
            $endgroup$
            – J.G.
            Apr 18 at 19:19










          • $begingroup$
            @AaronStevens: How do you address the fact that the photoelectric effect doesn't make sense if you consider light as a wave though? Could you explain why you think this?
            $endgroup$
            – Ben Crowell
            Apr 18 at 21:48










          • $begingroup$
            @BenCrowell I agree that QFT resolves this. I was talking more along the lines of classic EM waves where you would expect the intensity of the light to influence the energy of the ejected electrons and the frequency to influence the rate at which electrons are ejected, yet we see the reverse of this.
            $endgroup$
            – Aaron Stevens
            Apr 18 at 22:44











          • $begingroup$
            I'm guessing this is where the OP's confusion lies. We have some phenomena where the wave picture of light is sufficient to explain what's going on, but other times it doesn't work at all. When you are first learning it all it seems rather arbitrary without any deeper structure, where you just have to remember which picture is right to use depending on the system.
            $endgroup$
            – Aaron Stevens
            Apr 18 at 22:49















          8












          $begingroup$


          How does light 'choose' where to be a wave and where to be a particle?




          It doesn't. It always behaves as a wave (obeying the principle of superposition), and it always behaves as a particle (particle number being quantized).



          It sounds like you may have been influenced by someone who told you that light behaves like a particle in some experiments, and like a wave in others. That's false.






          share|cite|improve this answer









          $endgroup$








          • 4




            $begingroup$
            How do you address the fact that the photoelectric effect doesn't make sense if you consider light as a wave though? I don't think it is fine to say it always behaves like a wave, unless you mean to say the only consistent wave behavior is superposition
            $endgroup$
            – Aaron Stevens
            Apr 18 at 15:40










          • $begingroup$
            @AaronStevens The photoelectric effect doesn't "disprove" a wave description. If I place a guitar on a table and dust settles on one string, which I then pluck, whether I displace no specks of dust, one of them or several depends on how hard I pluck. But what caused any such displacement? A wave transmitting energy down the string. It's just that in QM, this is also explicable in terms of individual photons' absorption. This is why I recommend learning QFT, which makes better sense of how the wave/particle aspects mesh together.
            $endgroup$
            – J.G.
            Apr 18 at 19:19










          • $begingroup$
            @AaronStevens: How do you address the fact that the photoelectric effect doesn't make sense if you consider light as a wave though? Could you explain why you think this?
            $endgroup$
            – Ben Crowell
            Apr 18 at 21:48










          • $begingroup$
            @BenCrowell I agree that QFT resolves this. I was talking more along the lines of classic EM waves where you would expect the intensity of the light to influence the energy of the ejected electrons and the frequency to influence the rate at which electrons are ejected, yet we see the reverse of this.
            $endgroup$
            – Aaron Stevens
            Apr 18 at 22:44











          • $begingroup$
            I'm guessing this is where the OP's confusion lies. We have some phenomena where the wave picture of light is sufficient to explain what's going on, but other times it doesn't work at all. When you are first learning it all it seems rather arbitrary without any deeper structure, where you just have to remember which picture is right to use depending on the system.
            $endgroup$
            – Aaron Stevens
            Apr 18 at 22:49













          8












          8








          8





          $begingroup$


          How does light 'choose' where to be a wave and where to be a particle?




          It doesn't. It always behaves as a wave (obeying the principle of superposition), and it always behaves as a particle (particle number being quantized).



          It sounds like you may have been influenced by someone who told you that light behaves like a particle in some experiments, and like a wave in others. That's false.






          share|cite|improve this answer









          $endgroup$




          How does light 'choose' where to be a wave and where to be a particle?




          It doesn't. It always behaves as a wave (obeying the principle of superposition), and it always behaves as a particle (particle number being quantized).



          It sounds like you may have been influenced by someone who told you that light behaves like a particle in some experiments, and like a wave in others. That's false.







          share|cite|improve this answer












          share|cite|improve this answer



          share|cite|improve this answer










          answered Apr 18 at 15:37









          Ben CrowellBen Crowell

          55k6165317




          55k6165317







          • 4




            $begingroup$
            How do you address the fact that the photoelectric effect doesn't make sense if you consider light as a wave though? I don't think it is fine to say it always behaves like a wave, unless you mean to say the only consistent wave behavior is superposition
            $endgroup$
            – Aaron Stevens
            Apr 18 at 15:40










          • $begingroup$
            @AaronStevens The photoelectric effect doesn't "disprove" a wave description. If I place a guitar on a table and dust settles on one string, which I then pluck, whether I displace no specks of dust, one of them or several depends on how hard I pluck. But what caused any such displacement? A wave transmitting energy down the string. It's just that in QM, this is also explicable in terms of individual photons' absorption. This is why I recommend learning QFT, which makes better sense of how the wave/particle aspects mesh together.
            $endgroup$
            – J.G.
            Apr 18 at 19:19










          • $begingroup$
            @AaronStevens: How do you address the fact that the photoelectric effect doesn't make sense if you consider light as a wave though? Could you explain why you think this?
            $endgroup$
            – Ben Crowell
            Apr 18 at 21:48










          • $begingroup$
            @BenCrowell I agree that QFT resolves this. I was talking more along the lines of classic EM waves where you would expect the intensity of the light to influence the energy of the ejected electrons and the frequency to influence the rate at which electrons are ejected, yet we see the reverse of this.
            $endgroup$
            – Aaron Stevens
            Apr 18 at 22:44











          • $begingroup$
            I'm guessing this is where the OP's confusion lies. We have some phenomena where the wave picture of light is sufficient to explain what's going on, but other times it doesn't work at all. When you are first learning it all it seems rather arbitrary without any deeper structure, where you just have to remember which picture is right to use depending on the system.
            $endgroup$
            – Aaron Stevens
            Apr 18 at 22:49












          • 4




            $begingroup$
            How do you address the fact that the photoelectric effect doesn't make sense if you consider light as a wave though? I don't think it is fine to say it always behaves like a wave, unless you mean to say the only consistent wave behavior is superposition
            $endgroup$
            – Aaron Stevens
            Apr 18 at 15:40










          • $begingroup$
            @AaronStevens The photoelectric effect doesn't "disprove" a wave description. If I place a guitar on a table and dust settles on one string, which I then pluck, whether I displace no specks of dust, one of them or several depends on how hard I pluck. But what caused any such displacement? A wave transmitting energy down the string. It's just that in QM, this is also explicable in terms of individual photons' absorption. This is why I recommend learning QFT, which makes better sense of how the wave/particle aspects mesh together.
            $endgroup$
            – J.G.
            Apr 18 at 19:19










          • $begingroup$
            @AaronStevens: How do you address the fact that the photoelectric effect doesn't make sense if you consider light as a wave though? Could you explain why you think this?
            $endgroup$
            – Ben Crowell
            Apr 18 at 21:48










          • $begingroup$
            @BenCrowell I agree that QFT resolves this. I was talking more along the lines of classic EM waves where you would expect the intensity of the light to influence the energy of the ejected electrons and the frequency to influence the rate at which electrons are ejected, yet we see the reverse of this.
            $endgroup$
            – Aaron Stevens
            Apr 18 at 22:44











          • $begingroup$
            I'm guessing this is where the OP's confusion lies. We have some phenomena where the wave picture of light is sufficient to explain what's going on, but other times it doesn't work at all. When you are first learning it all it seems rather arbitrary without any deeper structure, where you just have to remember which picture is right to use depending on the system.
            $endgroup$
            – Aaron Stevens
            Apr 18 at 22:49







          4




          4




          $begingroup$
          How do you address the fact that the photoelectric effect doesn't make sense if you consider light as a wave though? I don't think it is fine to say it always behaves like a wave, unless you mean to say the only consistent wave behavior is superposition
          $endgroup$
          – Aaron Stevens
          Apr 18 at 15:40




          $begingroup$
          How do you address the fact that the photoelectric effect doesn't make sense if you consider light as a wave though? I don't think it is fine to say it always behaves like a wave, unless you mean to say the only consistent wave behavior is superposition
          $endgroup$
          – Aaron Stevens
          Apr 18 at 15:40












          $begingroup$
          @AaronStevens The photoelectric effect doesn't "disprove" a wave description. If I place a guitar on a table and dust settles on one string, which I then pluck, whether I displace no specks of dust, one of them or several depends on how hard I pluck. But what caused any such displacement? A wave transmitting energy down the string. It's just that in QM, this is also explicable in terms of individual photons' absorption. This is why I recommend learning QFT, which makes better sense of how the wave/particle aspects mesh together.
          $endgroup$
          – J.G.
          Apr 18 at 19:19




          $begingroup$
          @AaronStevens The photoelectric effect doesn't "disprove" a wave description. If I place a guitar on a table and dust settles on one string, which I then pluck, whether I displace no specks of dust, one of them or several depends on how hard I pluck. But what caused any such displacement? A wave transmitting energy down the string. It's just that in QM, this is also explicable in terms of individual photons' absorption. This is why I recommend learning QFT, which makes better sense of how the wave/particle aspects mesh together.
          $endgroup$
          – J.G.
          Apr 18 at 19:19












          $begingroup$
          @AaronStevens: How do you address the fact that the photoelectric effect doesn't make sense if you consider light as a wave though? Could you explain why you think this?
          $endgroup$
          – Ben Crowell
          Apr 18 at 21:48




          $begingroup$
          @AaronStevens: How do you address the fact that the photoelectric effect doesn't make sense if you consider light as a wave though? Could you explain why you think this?
          $endgroup$
          – Ben Crowell
          Apr 18 at 21:48












          $begingroup$
          @BenCrowell I agree that QFT resolves this. I was talking more along the lines of classic EM waves where you would expect the intensity of the light to influence the energy of the ejected electrons and the frequency to influence the rate at which electrons are ejected, yet we see the reverse of this.
          $endgroup$
          – Aaron Stevens
          Apr 18 at 22:44





          $begingroup$
          @BenCrowell I agree that QFT resolves this. I was talking more along the lines of classic EM waves where you would expect the intensity of the light to influence the energy of the ejected electrons and the frequency to influence the rate at which electrons are ejected, yet we see the reverse of this.
          $endgroup$
          – Aaron Stevens
          Apr 18 at 22:44













          $begingroup$
          I'm guessing this is where the OP's confusion lies. We have some phenomena where the wave picture of light is sufficient to explain what's going on, but other times it doesn't work at all. When you are first learning it all it seems rather arbitrary without any deeper structure, where you just have to remember which picture is right to use depending on the system.
          $endgroup$
          – Aaron Stevens
          Apr 18 at 22:49




          $begingroup$
          I'm guessing this is where the OP's confusion lies. We have some phenomena where the wave picture of light is sufficient to explain what's going on, but other times it doesn't work at all. When you are first learning it all it seems rather arbitrary without any deeper structure, where you just have to remember which picture is right to use depending on the system.
          $endgroup$
          – Aaron Stevens
          Apr 18 at 22:49











          5












          $begingroup$

          Light always behaves as a particle and waves .So there is no particular time when it can behaves like a particle but not wave and vice versa .Thus light carries both of these two nature(particle and wave) along with it (light)for all time.
          Actually why is my thinking is like light has two of these nature for all time?



          Reasons for considering it as *wave *



          1. James Clerk Maxwell proved that speed of the electromagnetic waves in free space is the same as the speed of light c=2.998 ×10^8m/s. Thereby he concluded that light consists of electromagnetic waves .So it has wave nature for all time.

          2. Light also shows *interference *,*diffraction *,*polarization *.
            And all of these property show that light has wave nature and it is for all time.

          Reasons for considering light as particle



          1. Max plank in 1900 introduced concept of quantum of energy .The energy exchange between radiation and surroundings is in *discrete or *quantised * form i.e energy exchange between electromagnetic waves is integer multiple of (plank constant *frequency of wave ).As light consists of electromagnetic waves then light is made of discrete bits of energy i.e photons with energy (plank constant *frequency of light).
            Thereby photoelectric effect ,Compton effect all show the particle nature of light.

          So radiation (electromagnetic waves) exhibit particle nature conversely material particles display wave like nature introduced by de Broglie(a moving particle has wave properties associated with it).This is confirmed experimentally by Davisson and Germer they proved that interference which is a property of waves can be obtained by with material particles such as electron.



          So as radiation exhibit particle like nature but particles also exhibit
          wave- like behavior .Thereby light always shows two type of natures both as waves and particles .






          share|cite|improve this answer









          $endgroup$












          • $begingroup$
            Ok if we say light is a particle and wave at all times, then why does it exhibit wave properties in certain cases and particle behaviour in certain phenomenon? Is it because it acts more like a wave and more like a particle in certain conditions? If so what are those conditions? Appreciate your answer btw
            $endgroup$
            – d_g
            Apr 18 at 17:58







          • 1




            $begingroup$
            @d_g light as a whole shows two natures as i said. you can consider double silt experiment when light passes through the silt it behaves like a wave but when it strikes the screen it behaves like a particle .
            $endgroup$
            – Sneha Banerjee
            Apr 18 at 19:04










          • $begingroup$
            Sounds good at 1st glance (as do the same general assertions from other respondents) BUT the single/double slit example can see "observed behaviour" change between apparent modes when the slit is opened/closed with timing such that behaviour "change" occurs with information transfer faster than C. ie a photon (or eg electron etc) ALWAYS "knows" whether the 'slit B' is open or closed at the time that it transits slit A and/or when it arrives at the target surface, even when the time taken travelling at C from slit B to slit A or target is > than the time that the photon etc takes to arrive.
            $endgroup$
            – Russell McMahon
            Apr 21 at 2:43















          5












          $begingroup$

          Light always behaves as a particle and waves .So there is no particular time when it can behaves like a particle but not wave and vice versa .Thus light carries both of these two nature(particle and wave) along with it (light)for all time.
          Actually why is my thinking is like light has two of these nature for all time?



          Reasons for considering it as *wave *



          1. James Clerk Maxwell proved that speed of the electromagnetic waves in free space is the same as the speed of light c=2.998 ×10^8m/s. Thereby he concluded that light consists of electromagnetic waves .So it has wave nature for all time.

          2. Light also shows *interference *,*diffraction *,*polarization *.
            And all of these property show that light has wave nature and it is for all time.

          Reasons for considering light as particle



          1. Max plank in 1900 introduced concept of quantum of energy .The energy exchange between radiation and surroundings is in *discrete or *quantised * form i.e energy exchange between electromagnetic waves is integer multiple of (plank constant *frequency of wave ).As light consists of electromagnetic waves then light is made of discrete bits of energy i.e photons with energy (plank constant *frequency of light).
            Thereby photoelectric effect ,Compton effect all show the particle nature of light.

          So radiation (electromagnetic waves) exhibit particle nature conversely material particles display wave like nature introduced by de Broglie(a moving particle has wave properties associated with it).This is confirmed experimentally by Davisson and Germer they proved that interference which is a property of waves can be obtained by with material particles such as electron.



          So as radiation exhibit particle like nature but particles also exhibit
          wave- like behavior .Thereby light always shows two type of natures both as waves and particles .






          share|cite|improve this answer









          $endgroup$












          • $begingroup$
            Ok if we say light is a particle and wave at all times, then why does it exhibit wave properties in certain cases and particle behaviour in certain phenomenon? Is it because it acts more like a wave and more like a particle in certain conditions? If so what are those conditions? Appreciate your answer btw
            $endgroup$
            – d_g
            Apr 18 at 17:58







          • 1




            $begingroup$
            @d_g light as a whole shows two natures as i said. you can consider double silt experiment when light passes through the silt it behaves like a wave but when it strikes the screen it behaves like a particle .
            $endgroup$
            – Sneha Banerjee
            Apr 18 at 19:04










          • $begingroup$
            Sounds good at 1st glance (as do the same general assertions from other respondents) BUT the single/double slit example can see "observed behaviour" change between apparent modes when the slit is opened/closed with timing such that behaviour "change" occurs with information transfer faster than C. ie a photon (or eg electron etc) ALWAYS "knows" whether the 'slit B' is open or closed at the time that it transits slit A and/or when it arrives at the target surface, even when the time taken travelling at C from slit B to slit A or target is > than the time that the photon etc takes to arrive.
            $endgroup$
            – Russell McMahon
            Apr 21 at 2:43













          5












          5








          5





          $begingroup$

          Light always behaves as a particle and waves .So there is no particular time when it can behaves like a particle but not wave and vice versa .Thus light carries both of these two nature(particle and wave) along with it (light)for all time.
          Actually why is my thinking is like light has two of these nature for all time?



          Reasons for considering it as *wave *



          1. James Clerk Maxwell proved that speed of the electromagnetic waves in free space is the same as the speed of light c=2.998 ×10^8m/s. Thereby he concluded that light consists of electromagnetic waves .So it has wave nature for all time.

          2. Light also shows *interference *,*diffraction *,*polarization *.
            And all of these property show that light has wave nature and it is for all time.

          Reasons for considering light as particle



          1. Max plank in 1900 introduced concept of quantum of energy .The energy exchange between radiation and surroundings is in *discrete or *quantised * form i.e energy exchange between electromagnetic waves is integer multiple of (plank constant *frequency of wave ).As light consists of electromagnetic waves then light is made of discrete bits of energy i.e photons with energy (plank constant *frequency of light).
            Thereby photoelectric effect ,Compton effect all show the particle nature of light.

          So radiation (electromagnetic waves) exhibit particle nature conversely material particles display wave like nature introduced by de Broglie(a moving particle has wave properties associated with it).This is confirmed experimentally by Davisson and Germer they proved that interference which is a property of waves can be obtained by with material particles such as electron.



          So as radiation exhibit particle like nature but particles also exhibit
          wave- like behavior .Thereby light always shows two type of natures both as waves and particles .






          share|cite|improve this answer









          $endgroup$



          Light always behaves as a particle and waves .So there is no particular time when it can behaves like a particle but not wave and vice versa .Thus light carries both of these two nature(particle and wave) along with it (light)for all time.
          Actually why is my thinking is like light has two of these nature for all time?



          Reasons for considering it as *wave *



          1. James Clerk Maxwell proved that speed of the electromagnetic waves in free space is the same as the speed of light c=2.998 ×10^8m/s. Thereby he concluded that light consists of electromagnetic waves .So it has wave nature for all time.

          2. Light also shows *interference *,*diffraction *,*polarization *.
            And all of these property show that light has wave nature and it is for all time.

          Reasons for considering light as particle



          1. Max plank in 1900 introduced concept of quantum of energy .The energy exchange between radiation and surroundings is in *discrete or *quantised * form i.e energy exchange between electromagnetic waves is integer multiple of (plank constant *frequency of wave ).As light consists of electromagnetic waves then light is made of discrete bits of energy i.e photons with energy (plank constant *frequency of light).
            Thereby photoelectric effect ,Compton effect all show the particle nature of light.

          So radiation (electromagnetic waves) exhibit particle nature conversely material particles display wave like nature introduced by de Broglie(a moving particle has wave properties associated with it).This is confirmed experimentally by Davisson and Germer they proved that interference which is a property of waves can be obtained by with material particles such as electron.



          So as radiation exhibit particle like nature but particles also exhibit
          wave- like behavior .Thereby light always shows two type of natures both as waves and particles .







          share|cite|improve this answer












          share|cite|improve this answer



          share|cite|improve this answer










          answered Apr 18 at 17:34









          Sneha BanerjeeSneha Banerjee

          645




          645











          • $begingroup$
            Ok if we say light is a particle and wave at all times, then why does it exhibit wave properties in certain cases and particle behaviour in certain phenomenon? Is it because it acts more like a wave and more like a particle in certain conditions? If so what are those conditions? Appreciate your answer btw
            $endgroup$
            – d_g
            Apr 18 at 17:58







          • 1




            $begingroup$
            @d_g light as a whole shows two natures as i said. you can consider double silt experiment when light passes through the silt it behaves like a wave but when it strikes the screen it behaves like a particle .
            $endgroup$
            – Sneha Banerjee
            Apr 18 at 19:04










          • $begingroup$
            Sounds good at 1st glance (as do the same general assertions from other respondents) BUT the single/double slit example can see "observed behaviour" change between apparent modes when the slit is opened/closed with timing such that behaviour "change" occurs with information transfer faster than C. ie a photon (or eg electron etc) ALWAYS "knows" whether the 'slit B' is open or closed at the time that it transits slit A and/or when it arrives at the target surface, even when the time taken travelling at C from slit B to slit A or target is > than the time that the photon etc takes to arrive.
            $endgroup$
            – Russell McMahon
            Apr 21 at 2:43
















          • $begingroup$
            Ok if we say light is a particle and wave at all times, then why does it exhibit wave properties in certain cases and particle behaviour in certain phenomenon? Is it because it acts more like a wave and more like a particle in certain conditions? If so what are those conditions? Appreciate your answer btw
            $endgroup$
            – d_g
            Apr 18 at 17:58







          • 1




            $begingroup$
            @d_g light as a whole shows two natures as i said. you can consider double silt experiment when light passes through the silt it behaves like a wave but when it strikes the screen it behaves like a particle .
            $endgroup$
            – Sneha Banerjee
            Apr 18 at 19:04










          • $begingroup$
            Sounds good at 1st glance (as do the same general assertions from other respondents) BUT the single/double slit example can see "observed behaviour" change between apparent modes when the slit is opened/closed with timing such that behaviour "change" occurs with information transfer faster than C. ie a photon (or eg electron etc) ALWAYS "knows" whether the 'slit B' is open or closed at the time that it transits slit A and/or when it arrives at the target surface, even when the time taken travelling at C from slit B to slit A or target is > than the time that the photon etc takes to arrive.
            $endgroup$
            – Russell McMahon
            Apr 21 at 2:43















          $begingroup$
          Ok if we say light is a particle and wave at all times, then why does it exhibit wave properties in certain cases and particle behaviour in certain phenomenon? Is it because it acts more like a wave and more like a particle in certain conditions? If so what are those conditions? Appreciate your answer btw
          $endgroup$
          – d_g
          Apr 18 at 17:58





          $begingroup$
          Ok if we say light is a particle and wave at all times, then why does it exhibit wave properties in certain cases and particle behaviour in certain phenomenon? Is it because it acts more like a wave and more like a particle in certain conditions? If so what are those conditions? Appreciate your answer btw
          $endgroup$
          – d_g
          Apr 18 at 17:58





          1




          1




          $begingroup$
          @d_g light as a whole shows two natures as i said. you can consider double silt experiment when light passes through the silt it behaves like a wave but when it strikes the screen it behaves like a particle .
          $endgroup$
          – Sneha Banerjee
          Apr 18 at 19:04




          $begingroup$
          @d_g light as a whole shows two natures as i said. you can consider double silt experiment when light passes through the silt it behaves like a wave but when it strikes the screen it behaves like a particle .
          $endgroup$
          – Sneha Banerjee
          Apr 18 at 19:04












          $begingroup$
          Sounds good at 1st glance (as do the same general assertions from other respondents) BUT the single/double slit example can see "observed behaviour" change between apparent modes when the slit is opened/closed with timing such that behaviour "change" occurs with information transfer faster than C. ie a photon (or eg electron etc) ALWAYS "knows" whether the 'slit B' is open or closed at the time that it transits slit A and/or when it arrives at the target surface, even when the time taken travelling at C from slit B to slit A or target is > than the time that the photon etc takes to arrive.
          $endgroup$
          – Russell McMahon
          Apr 21 at 2:43




          $begingroup$
          Sounds good at 1st glance (as do the same general assertions from other respondents) BUT the single/double slit example can see "observed behaviour" change between apparent modes when the slit is opened/closed with timing such that behaviour "change" occurs with information transfer faster than C. ie a photon (or eg electron etc) ALWAYS "knows" whether the 'slit B' is open or closed at the time that it transits slit A and/or when it arrives at the target surface, even when the time taken travelling at C from slit B to slit A or target is > than the time that the photon etc takes to arrive.
          $endgroup$
          – Russell McMahon
          Apr 21 at 2:43











          4












          $begingroup$

          The fundamental experiment showing the apparent contradiction is Young's double slit: How can particle characteristics be transmitted when there is only an interfering wave between the point A of emission and point B of absorption?



          However, for photons in vacuum (moving at c) there is a simple answer: The spacetime interval between A and B is empty, it is zero! That means that both points A and B are adjacent. A and B may be represented by mass particles (electrons etc.) which are exchanging a momentum. The transmission is direct, without need of any intermediate particle.



          In contrast, a spacetime interval cannot be observed by observers. If a light ray is transmitted from Sun to Earth, nobody will see that A (Sun) and B (Earth) are adjacent. Instead, they will observe a space distance of eight light minutes and a time interval of eight minutes, even if the spacetime interval is zero. In this situation, the light wave takes the role of a sort of "placeholder": Light waves are observed to propagate at c (according to the second postulate of special relativity), but this is mere observation.



          In short, the particle characteristics may be transmitted without any photon because the spacetime interval is zero. The wave characteristics (including the propagation at c) are observation only.



          By the way, for light propagating at a lower speed than c (e.g. light moving through a medium), we need quantum mechanics for the answer.






          share|cite|improve this answer









          $endgroup$












          • $begingroup$
            Double slit works for eg electrons.
            $endgroup$
            – Russell McMahon
            Apr 20 at 21:29










          • $begingroup$
            I like you acknowledgement that all photons huddle together in a timeless spaceless group nattering together (presumably) and waiting the end of all things :-). I've long loved and pondered this aspect of reality and wondered if we can ever properly appreciate its meaning :-). || I wrote my one liner above just before rushing off to Easter Sunday church service. I've just reread it at further below write-speed and note now your assertions relating to A & B being able to be massed particles but the communications being mediated not at C but instantaneously ...
            $endgroup$
            – Russell McMahon
            Apr 21 at 2:33










          • $begingroup$
            ... (or at least > or >> C, as is seen to be the case experimentally). I don't recall seeing that asserted anywhere previously (this is not a specialist area of mine). It doesn't SEEM right but it certainly makes sense of the results :-). Do you have any references for that?
            $endgroup$
            – Russell McMahon
            Apr 21 at 2:35










          • $begingroup$
            @Russell McMahon, the transmission of the momentum is not instantaneous, in the Sun-Earth example 8 minutes, and the distance is still 8 light minutes. I simply emphasize the fact that the spacetime interval is zero, and that means that from this point of view A and B are adjacent such that there is a possibility for transmission of particle characteristics. In contrast, quantum mechanics is needed when mass is concerned (light through medium, wave-particle duality of mass particles such as electrons).
            $endgroup$
            – Moonraker
            Apr 21 at 5:37











          • $begingroup$
            You might also be interested in this question which did not receive a satisfying answer yet. Happy Easter!
            $endgroup$
            – Moonraker
            Apr 21 at 5:39















          4












          $begingroup$

          The fundamental experiment showing the apparent contradiction is Young's double slit: How can particle characteristics be transmitted when there is only an interfering wave between the point A of emission and point B of absorption?



          However, for photons in vacuum (moving at c) there is a simple answer: The spacetime interval between A and B is empty, it is zero! That means that both points A and B are adjacent. A and B may be represented by mass particles (electrons etc.) which are exchanging a momentum. The transmission is direct, without need of any intermediate particle.



          In contrast, a spacetime interval cannot be observed by observers. If a light ray is transmitted from Sun to Earth, nobody will see that A (Sun) and B (Earth) are adjacent. Instead, they will observe a space distance of eight light minutes and a time interval of eight minutes, even if the spacetime interval is zero. In this situation, the light wave takes the role of a sort of "placeholder": Light waves are observed to propagate at c (according to the second postulate of special relativity), but this is mere observation.



          In short, the particle characteristics may be transmitted without any photon because the spacetime interval is zero. The wave characteristics (including the propagation at c) are observation only.



          By the way, for light propagating at a lower speed than c (e.g. light moving through a medium), we need quantum mechanics for the answer.






          share|cite|improve this answer









          $endgroup$












          • $begingroup$
            Double slit works for eg electrons.
            $endgroup$
            – Russell McMahon
            Apr 20 at 21:29










          • $begingroup$
            I like you acknowledgement that all photons huddle together in a timeless spaceless group nattering together (presumably) and waiting the end of all things :-). I've long loved and pondered this aspect of reality and wondered if we can ever properly appreciate its meaning :-). || I wrote my one liner above just before rushing off to Easter Sunday church service. I've just reread it at further below write-speed and note now your assertions relating to A & B being able to be massed particles but the communications being mediated not at C but instantaneously ...
            $endgroup$
            – Russell McMahon
            Apr 21 at 2:33










          • $begingroup$
            ... (or at least > or >> C, as is seen to be the case experimentally). I don't recall seeing that asserted anywhere previously (this is not a specialist area of mine). It doesn't SEEM right but it certainly makes sense of the results :-). Do you have any references for that?
            $endgroup$
            – Russell McMahon
            Apr 21 at 2:35










          • $begingroup$
            @Russell McMahon, the transmission of the momentum is not instantaneous, in the Sun-Earth example 8 minutes, and the distance is still 8 light minutes. I simply emphasize the fact that the spacetime interval is zero, and that means that from this point of view A and B are adjacent such that there is a possibility for transmission of particle characteristics. In contrast, quantum mechanics is needed when mass is concerned (light through medium, wave-particle duality of mass particles such as electrons).
            $endgroup$
            – Moonraker
            Apr 21 at 5:37











          • $begingroup$
            You might also be interested in this question which did not receive a satisfying answer yet. Happy Easter!
            $endgroup$
            – Moonraker
            Apr 21 at 5:39













          4












          4








          4





          $begingroup$

          The fundamental experiment showing the apparent contradiction is Young's double slit: How can particle characteristics be transmitted when there is only an interfering wave between the point A of emission and point B of absorption?



          However, for photons in vacuum (moving at c) there is a simple answer: The spacetime interval between A and B is empty, it is zero! That means that both points A and B are adjacent. A and B may be represented by mass particles (electrons etc.) which are exchanging a momentum. The transmission is direct, without need of any intermediate particle.



          In contrast, a spacetime interval cannot be observed by observers. If a light ray is transmitted from Sun to Earth, nobody will see that A (Sun) and B (Earth) are adjacent. Instead, they will observe a space distance of eight light minutes and a time interval of eight minutes, even if the spacetime interval is zero. In this situation, the light wave takes the role of a sort of "placeholder": Light waves are observed to propagate at c (according to the second postulate of special relativity), but this is mere observation.



          In short, the particle characteristics may be transmitted without any photon because the spacetime interval is zero. The wave characteristics (including the propagation at c) are observation only.



          By the way, for light propagating at a lower speed than c (e.g. light moving through a medium), we need quantum mechanics for the answer.






          share|cite|improve this answer









          $endgroup$



          The fundamental experiment showing the apparent contradiction is Young's double slit: How can particle characteristics be transmitted when there is only an interfering wave between the point A of emission and point B of absorption?



          However, for photons in vacuum (moving at c) there is a simple answer: The spacetime interval between A and B is empty, it is zero! That means that both points A and B are adjacent. A and B may be represented by mass particles (electrons etc.) which are exchanging a momentum. The transmission is direct, without need of any intermediate particle.



          In contrast, a spacetime interval cannot be observed by observers. If a light ray is transmitted from Sun to Earth, nobody will see that A (Sun) and B (Earth) are adjacent. Instead, they will observe a space distance of eight light minutes and a time interval of eight minutes, even if the spacetime interval is zero. In this situation, the light wave takes the role of a sort of "placeholder": Light waves are observed to propagate at c (according to the second postulate of special relativity), but this is mere observation.



          In short, the particle characteristics may be transmitted without any photon because the spacetime interval is zero. The wave characteristics (including the propagation at c) are observation only.



          By the way, for light propagating at a lower speed than c (e.g. light moving through a medium), we need quantum mechanics for the answer.







          share|cite|improve this answer












          share|cite|improve this answer



          share|cite|improve this answer










          answered Apr 18 at 17:00









          MoonrakerMoonraker

          1,99411023




          1,99411023











          • $begingroup$
            Double slit works for eg electrons.
            $endgroup$
            – Russell McMahon
            Apr 20 at 21:29










          • $begingroup$
            I like you acknowledgement that all photons huddle together in a timeless spaceless group nattering together (presumably) and waiting the end of all things :-). I've long loved and pondered this aspect of reality and wondered if we can ever properly appreciate its meaning :-). || I wrote my one liner above just before rushing off to Easter Sunday church service. I've just reread it at further below write-speed and note now your assertions relating to A & B being able to be massed particles but the communications being mediated not at C but instantaneously ...
            $endgroup$
            – Russell McMahon
            Apr 21 at 2:33










          • $begingroup$
            ... (or at least > or >> C, as is seen to be the case experimentally). I don't recall seeing that asserted anywhere previously (this is not a specialist area of mine). It doesn't SEEM right but it certainly makes sense of the results :-). Do you have any references for that?
            $endgroup$
            – Russell McMahon
            Apr 21 at 2:35










          • $begingroup$
            @Russell McMahon, the transmission of the momentum is not instantaneous, in the Sun-Earth example 8 minutes, and the distance is still 8 light minutes. I simply emphasize the fact that the spacetime interval is zero, and that means that from this point of view A and B are adjacent such that there is a possibility for transmission of particle characteristics. In contrast, quantum mechanics is needed when mass is concerned (light through medium, wave-particle duality of mass particles such as electrons).
            $endgroup$
            – Moonraker
            Apr 21 at 5:37











          • $begingroup$
            You might also be interested in this question which did not receive a satisfying answer yet. Happy Easter!
            $endgroup$
            – Moonraker
            Apr 21 at 5:39
















          • $begingroup$
            Double slit works for eg electrons.
            $endgroup$
            – Russell McMahon
            Apr 20 at 21:29










          • $begingroup$
            I like you acknowledgement that all photons huddle together in a timeless spaceless group nattering together (presumably) and waiting the end of all things :-). I've long loved and pondered this aspect of reality and wondered if we can ever properly appreciate its meaning :-). || I wrote my one liner above just before rushing off to Easter Sunday church service. I've just reread it at further below write-speed and note now your assertions relating to A & B being able to be massed particles but the communications being mediated not at C but instantaneously ...
            $endgroup$
            – Russell McMahon
            Apr 21 at 2:33










          • $begingroup$
            ... (or at least > or >> C, as is seen to be the case experimentally). I don't recall seeing that asserted anywhere previously (this is not a specialist area of mine). It doesn't SEEM right but it certainly makes sense of the results :-). Do you have any references for that?
            $endgroup$
            – Russell McMahon
            Apr 21 at 2:35










          • $begingroup$
            @Russell McMahon, the transmission of the momentum is not instantaneous, in the Sun-Earth example 8 minutes, and the distance is still 8 light minutes. I simply emphasize the fact that the spacetime interval is zero, and that means that from this point of view A and B are adjacent such that there is a possibility for transmission of particle characteristics. In contrast, quantum mechanics is needed when mass is concerned (light through medium, wave-particle duality of mass particles such as electrons).
            $endgroup$
            – Moonraker
            Apr 21 at 5:37











          • $begingroup$
            You might also be interested in this question which did not receive a satisfying answer yet. Happy Easter!
            $endgroup$
            – Moonraker
            Apr 21 at 5:39















          $begingroup$
          Double slit works for eg electrons.
          $endgroup$
          – Russell McMahon
          Apr 20 at 21:29




          $begingroup$
          Double slit works for eg electrons.
          $endgroup$
          – Russell McMahon
          Apr 20 at 21:29












          $begingroup$
          I like you acknowledgement that all photons huddle together in a timeless spaceless group nattering together (presumably) and waiting the end of all things :-). I've long loved and pondered this aspect of reality and wondered if we can ever properly appreciate its meaning :-). || I wrote my one liner above just before rushing off to Easter Sunday church service. I've just reread it at further below write-speed and note now your assertions relating to A & B being able to be massed particles but the communications being mediated not at C but instantaneously ...
          $endgroup$
          – Russell McMahon
          Apr 21 at 2:33




          $begingroup$
          I like you acknowledgement that all photons huddle together in a timeless spaceless group nattering together (presumably) and waiting the end of all things :-). I've long loved and pondered this aspect of reality and wondered if we can ever properly appreciate its meaning :-). || I wrote my one liner above just before rushing off to Easter Sunday church service. I've just reread it at further below write-speed and note now your assertions relating to A & B being able to be massed particles but the communications being mediated not at C but instantaneously ...
          $endgroup$
          – Russell McMahon
          Apr 21 at 2:33












          $begingroup$
          ... (or at least > or >> C, as is seen to be the case experimentally). I don't recall seeing that asserted anywhere previously (this is not a specialist area of mine). It doesn't SEEM right but it certainly makes sense of the results :-). Do you have any references for that?
          $endgroup$
          – Russell McMahon
          Apr 21 at 2:35




          $begingroup$
          ... (or at least > or >> C, as is seen to be the case experimentally). I don't recall seeing that asserted anywhere previously (this is not a specialist area of mine). It doesn't SEEM right but it certainly makes sense of the results :-). Do you have any references for that?
          $endgroup$
          – Russell McMahon
          Apr 21 at 2:35












          $begingroup$
          @Russell McMahon, the transmission of the momentum is not instantaneous, in the Sun-Earth example 8 minutes, and the distance is still 8 light minutes. I simply emphasize the fact that the spacetime interval is zero, and that means that from this point of view A and B are adjacent such that there is a possibility for transmission of particle characteristics. In contrast, quantum mechanics is needed when mass is concerned (light through medium, wave-particle duality of mass particles such as electrons).
          $endgroup$
          – Moonraker
          Apr 21 at 5:37





          $begingroup$
          @Russell McMahon, the transmission of the momentum is not instantaneous, in the Sun-Earth example 8 minutes, and the distance is still 8 light minutes. I simply emphasize the fact that the spacetime interval is zero, and that means that from this point of view A and B are adjacent such that there is a possibility for transmission of particle characteristics. In contrast, quantum mechanics is needed when mass is concerned (light through medium, wave-particle duality of mass particles such as electrons).
          $endgroup$
          – Moonraker
          Apr 21 at 5:37













          $begingroup$
          You might also be interested in this question which did not receive a satisfying answer yet. Happy Easter!
          $endgroup$
          – Moonraker
          Apr 21 at 5:39




          $begingroup$
          You might also be interested in this question which did not receive a satisfying answer yet. Happy Easter!
          $endgroup$
          – Moonraker
          Apr 21 at 5:39











          4












          $begingroup$

          The light doesn't chose. You as an experimenter chose which observable you want to measure, and thus which operator you use. Such measurement will result in a wavefunction collapsing into one of the eigenstates of this operator.
          E.g. a positiin operator will give you a position, i.e. particle.
          A momentum operator will give you a momentum, i.e. wavelike object.






          share|cite|improve this answer








          New contributor




          Alex Vozny is a new contributor to this site. Take care in asking for clarification, commenting, and answering.
          Check out our Code of Conduct.






          $endgroup$












          • $begingroup$
            No. See my comment to @Sneha's answer. In eg the single/double slit experiment the observation is the same and opening/closing a second slit "at a distance" alters the observed result - even when a single eg photon is used and the information transfer occurs faster than C. The photon "KNOWS" what is happening at distances far enough away that no information can 'tell it' so in classical terms. || Your answer is essentially saying that the observers behaviour changes when eg a slit is opened or closed and the setup-change causes the observer to change behaviour. This MAY be true :-) !
            $endgroup$
            – Russell McMahon
            Apr 21 at 2:49















          4












          $begingroup$

          The light doesn't chose. You as an experimenter chose which observable you want to measure, and thus which operator you use. Such measurement will result in a wavefunction collapsing into one of the eigenstates of this operator.
          E.g. a positiin operator will give you a position, i.e. particle.
          A momentum operator will give you a momentum, i.e. wavelike object.






          share|cite|improve this answer








          New contributor




          Alex Vozny is a new contributor to this site. Take care in asking for clarification, commenting, and answering.
          Check out our Code of Conduct.






          $endgroup$












          • $begingroup$
            No. See my comment to @Sneha's answer. In eg the single/double slit experiment the observation is the same and opening/closing a second slit "at a distance" alters the observed result - even when a single eg photon is used and the information transfer occurs faster than C. The photon "KNOWS" what is happening at distances far enough away that no information can 'tell it' so in classical terms. || Your answer is essentially saying that the observers behaviour changes when eg a slit is opened or closed and the setup-change causes the observer to change behaviour. This MAY be true :-) !
            $endgroup$
            – Russell McMahon
            Apr 21 at 2:49













          4












          4








          4





          $begingroup$

          The light doesn't chose. You as an experimenter chose which observable you want to measure, and thus which operator you use. Such measurement will result in a wavefunction collapsing into one of the eigenstates of this operator.
          E.g. a positiin operator will give you a position, i.e. particle.
          A momentum operator will give you a momentum, i.e. wavelike object.






          share|cite|improve this answer








          New contributor




          Alex Vozny is a new contributor to this site. Take care in asking for clarification, commenting, and answering.
          Check out our Code of Conduct.






          $endgroup$



          The light doesn't chose. You as an experimenter chose which observable you want to measure, and thus which operator you use. Such measurement will result in a wavefunction collapsing into one of the eigenstates of this operator.
          E.g. a positiin operator will give you a position, i.e. particle.
          A momentum operator will give you a momentum, i.e. wavelike object.







          share|cite|improve this answer








          New contributor




          Alex Vozny is a new contributor to this site. Take care in asking for clarification, commenting, and answering.
          Check out our Code of Conduct.









          share|cite|improve this answer



          share|cite|improve this answer






          New contributor




          Alex Vozny is a new contributor to this site. Take care in asking for clarification, commenting, and answering.
          Check out our Code of Conduct.









          answered Apr 19 at 1:46









          Alex VoznyAlex Vozny

          411




          411




          New contributor




          Alex Vozny is a new contributor to this site. Take care in asking for clarification, commenting, and answering.
          Check out our Code of Conduct.





          New contributor





          Alex Vozny is a new contributor to this site. Take care in asking for clarification, commenting, and answering.
          Check out our Code of Conduct.






          Alex Vozny is a new contributor to this site. Take care in asking for clarification, commenting, and answering.
          Check out our Code of Conduct.











          • $begingroup$
            No. See my comment to @Sneha's answer. In eg the single/double slit experiment the observation is the same and opening/closing a second slit "at a distance" alters the observed result - even when a single eg photon is used and the information transfer occurs faster than C. The photon "KNOWS" what is happening at distances far enough away that no information can 'tell it' so in classical terms. || Your answer is essentially saying that the observers behaviour changes when eg a slit is opened or closed and the setup-change causes the observer to change behaviour. This MAY be true :-) !
            $endgroup$
            – Russell McMahon
            Apr 21 at 2:49
















          • $begingroup$
            No. See my comment to @Sneha's answer. In eg the single/double slit experiment the observation is the same and opening/closing a second slit "at a distance" alters the observed result - even when a single eg photon is used and the information transfer occurs faster than C. The photon "KNOWS" what is happening at distances far enough away that no information can 'tell it' so in classical terms. || Your answer is essentially saying that the observers behaviour changes when eg a slit is opened or closed and the setup-change causes the observer to change behaviour. This MAY be true :-) !
            $endgroup$
            – Russell McMahon
            Apr 21 at 2:49















          $begingroup$
          No. See my comment to @Sneha's answer. In eg the single/double slit experiment the observation is the same and opening/closing a second slit "at a distance" alters the observed result - even when a single eg photon is used and the information transfer occurs faster than C. The photon "KNOWS" what is happening at distances far enough away that no information can 'tell it' so in classical terms. || Your answer is essentially saying that the observers behaviour changes when eg a slit is opened or closed and the setup-change causes the observer to change behaviour. This MAY be true :-) !
          $endgroup$
          – Russell McMahon
          Apr 21 at 2:49




          $begingroup$
          No. See my comment to @Sneha's answer. In eg the single/double slit experiment the observation is the same and opening/closing a second slit "at a distance" alters the observed result - even when a single eg photon is used and the information transfer occurs faster than C. The photon "KNOWS" what is happening at distances far enough away that no information can 'tell it' so in classical terms. || Your answer is essentially saying that the observers behaviour changes when eg a slit is opened or closed and the setup-change causes the observer to change behaviour. This MAY be true :-) !
          $endgroup$
          – Russell McMahon
          Apr 21 at 2:49











          3












          $begingroup$

          Light is assumed to be particle or wave to describe the observed phenomenon. Light is what it is and doesn't choose. However, one can consider light to be a particle when the interaction is with matter & as a wave when it interacts with itself.






          share|cite|improve this answer








          New contributor




          navnth is a new contributor to this site. Take care in asking for clarification, commenting, and answering.
          Check out our Code of Conduct.






          $endgroup$

















            3












            $begingroup$

            Light is assumed to be particle or wave to describe the observed phenomenon. Light is what it is and doesn't choose. However, one can consider light to be a particle when the interaction is with matter & as a wave when it interacts with itself.






            share|cite|improve this answer








            New contributor




            navnth is a new contributor to this site. Take care in asking for clarification, commenting, and answering.
            Check out our Code of Conduct.






            $endgroup$















              3












              3








              3





              $begingroup$

              Light is assumed to be particle or wave to describe the observed phenomenon. Light is what it is and doesn't choose. However, one can consider light to be a particle when the interaction is with matter & as a wave when it interacts with itself.






              share|cite|improve this answer








              New contributor




              navnth is a new contributor to this site. Take care in asking for clarification, commenting, and answering.
              Check out our Code of Conduct.






              $endgroup$



              Light is assumed to be particle or wave to describe the observed phenomenon. Light is what it is and doesn't choose. However, one can consider light to be a particle when the interaction is with matter & as a wave when it interacts with itself.







              share|cite|improve this answer








              New contributor




              navnth is a new contributor to this site. Take care in asking for clarification, commenting, and answering.
              Check out our Code of Conduct.









              share|cite|improve this answer



              share|cite|improve this answer






              New contributor




              navnth is a new contributor to this site. Take care in asking for clarification, commenting, and answering.
              Check out our Code of Conduct.









              answered Apr 19 at 8:16









              navnthnavnth

              312




              312




              New contributor




              navnth is a new contributor to this site. Take care in asking for clarification, commenting, and answering.
              Check out our Code of Conduct.





              New contributor





              navnth is a new contributor to this site. Take care in asking for clarification, commenting, and answering.
              Check out our Code of Conduct.






              navnth is a new contributor to this site. Take care in asking for clarification, commenting, and answering.
              Check out our Code of Conduct.





















                  2












                  $begingroup$

                  It doesn't is the short answer, as it is really neither. Basically, waves and particles are things you see on the classical scale, they model behaviour on macroscopic scales.



                  Objects don't behave in a classical way any more when we shrink down to very small scales and they obey very different dynamics which seem strange to us because we do not live on that scale and are only able to access things on that scale in quite a dim way via theory and experiment. It's kind of common sense when you think about it, just stop expecting that things will behave in a classical way when you going down to those length scales.



                  Also forgot the wave-particle duality, as it does not even make sense. It's like a blind person touching an elephant on the trunk and then touching it again on the leg and saying that the elephant follows trunk-leg duality.






                  share|cite|improve this answer









                  $endgroup$

















                    2












                    $begingroup$

                    It doesn't is the short answer, as it is really neither. Basically, waves and particles are things you see on the classical scale, they model behaviour on macroscopic scales.



                    Objects don't behave in a classical way any more when we shrink down to very small scales and they obey very different dynamics which seem strange to us because we do not live on that scale and are only able to access things on that scale in quite a dim way via theory and experiment. It's kind of common sense when you think about it, just stop expecting that things will behave in a classical way when you going down to those length scales.



                    Also forgot the wave-particle duality, as it does not even make sense. It's like a blind person touching an elephant on the trunk and then touching it again on the leg and saying that the elephant follows trunk-leg duality.






                    share|cite|improve this answer









                    $endgroup$















                      2












                      2








                      2





                      $begingroup$

                      It doesn't is the short answer, as it is really neither. Basically, waves and particles are things you see on the classical scale, they model behaviour on macroscopic scales.



                      Objects don't behave in a classical way any more when we shrink down to very small scales and they obey very different dynamics which seem strange to us because we do not live on that scale and are only able to access things on that scale in quite a dim way via theory and experiment. It's kind of common sense when you think about it, just stop expecting that things will behave in a classical way when you going down to those length scales.



                      Also forgot the wave-particle duality, as it does not even make sense. It's like a blind person touching an elephant on the trunk and then touching it again on the leg and saying that the elephant follows trunk-leg duality.






                      share|cite|improve this answer









                      $endgroup$



                      It doesn't is the short answer, as it is really neither. Basically, waves and particles are things you see on the classical scale, they model behaviour on macroscopic scales.



                      Objects don't behave in a classical way any more when we shrink down to very small scales and they obey very different dynamics which seem strange to us because we do not live on that scale and are only able to access things on that scale in quite a dim way via theory and experiment. It's kind of common sense when you think about it, just stop expecting that things will behave in a classical way when you going down to those length scales.



                      Also forgot the wave-particle duality, as it does not even make sense. It's like a blind person touching an elephant on the trunk and then touching it again on the leg and saying that the elephant follows trunk-leg duality.







                      share|cite|improve this answer












                      share|cite|improve this answer



                      share|cite|improve this answer










                      answered Apr 19 at 16:54









                      TomTom

                      22819




                      22819





















                          2












                          $begingroup$

                          It doesn’t.



                          Light, as you may know, is made up of photons, and photons are particles. At the same time, all particles are waves. Yes, that's right—particles are not points or spheres or anything else really solid moving around, but waves, which interact with each other in various ways.



                          Now, these waves may sometimes behave indistinguishably from points moving around in space, and may therefore be approximated as such—what we like to refer to as point particles—but they are fundamentally still always waves.



                          So, a photon is always both a wave and a particle; these are not mutually exclusive (in fact, as I have explained now, particles are a subset of waves). This is what is known as the wave–particle duality.






                          share|cite|improve this answer











                          $endgroup$

















                            2












                            $begingroup$

                            It doesn’t.



                            Light, as you may know, is made up of photons, and photons are particles. At the same time, all particles are waves. Yes, that's right—particles are not points or spheres or anything else really solid moving around, but waves, which interact with each other in various ways.



                            Now, these waves may sometimes behave indistinguishably from points moving around in space, and may therefore be approximated as such—what we like to refer to as point particles—but they are fundamentally still always waves.



                            So, a photon is always both a wave and a particle; these are not mutually exclusive (in fact, as I have explained now, particles are a subset of waves). This is what is known as the wave–particle duality.






                            share|cite|improve this answer











                            $endgroup$















                              2












                              2








                              2





                              $begingroup$

                              It doesn’t.



                              Light, as you may know, is made up of photons, and photons are particles. At the same time, all particles are waves. Yes, that's right—particles are not points or spheres or anything else really solid moving around, but waves, which interact with each other in various ways.



                              Now, these waves may sometimes behave indistinguishably from points moving around in space, and may therefore be approximated as such—what we like to refer to as point particles—but they are fundamentally still always waves.



                              So, a photon is always both a wave and a particle; these are not mutually exclusive (in fact, as I have explained now, particles are a subset of waves). This is what is known as the wave–particle duality.






                              share|cite|improve this answer











                              $endgroup$



                              It doesn’t.



                              Light, as you may know, is made up of photons, and photons are particles. At the same time, all particles are waves. Yes, that's right—particles are not points or spheres or anything else really solid moving around, but waves, which interact with each other in various ways.



                              Now, these waves may sometimes behave indistinguishably from points moving around in space, and may therefore be approximated as such—what we like to refer to as point particles—but they are fundamentally still always waves.



                              So, a photon is always both a wave and a particle; these are not mutually exclusive (in fact, as I have explained now, particles are a subset of waves). This is what is known as the wave–particle duality.







                              share|cite|improve this answer














                              share|cite|improve this answer



                              share|cite|improve this answer








                              edited Apr 19 at 19:23

























                              answered Apr 19 at 13:35









                              HelloGoodbyeHelloGoodbye

                              207127




                              207127





















                                  1












                                  $begingroup$

                                  binaryfunt and Ben Crowell's answer is closest to the truth. Let me add a few things.



                                  You get confused because you read sentences like, light behaves as a wave when it travels in space and as a particle when it interferes with matter. That is not true. In reality light has characteristics of both waves and particles at the same time. The truth is, we do not know what actually light is, or what it is really made of.



                                  In reality, light is not what decides or behaves different ways, it is us, the ones preparing the experiments, who decide whether we will select from the observed (both wave or particle like) nature the more interesting wave or particle like characteristics, and analyze those for certain purposes.



                                  What we try to do, is model with our mathematical ways the real phenomenon in our physical world, light itself.



                                  The reason you get confused is because there are two ways to really model this phenomenon:



                                  1. classical EM waves


                                  2. QM particles, photons


                                  You would think they do not match, but that is not true. In reality the two methods come together perfectly, as we build up the classical EM waves from herds of QM photons.



                                  In our currently accepted theory, the Standard Model, we talk about elementary particles, that have no internal structure, nor spatial extent, and we call them point like particles. One of these particles is the photon. We call it the quanta of the EM wave.



                                  It is very important to see that the basic confusion lies in whether you want to analyze the wave or the particle characteristics of light in a certain experiment.



                                  We usually model light in two ways, and that is what confuses you, these to ways are whether it travels or when it interacts with matter. This is basically a misconception, because light could be analyzed to show wave and particles characteristics both when it travels and when it interacts with matter.



                                  It comes from a misconception to choose between travel and interaction, but for the sake of argument, lets see:



                                  1. travel

                                  You can use Maxwell's original equations or use QFT to treat photons and light as excitation of the EM field, that travels through space, and this propagation is modeled in our theories as a wave. This is because this wave model is what best fits the experimental data. Now we can show that the QM or QFT field excitation model of the photon propagating through space is the same fitting the data perfectly from the experiments.



                                  So you see light has both wave and particle characteristics even as it travels through space.



                                  1. interaction with matter

                                  Now when a photon interacts with an atom, three things can happen:



                                  1. elastic scattering, the photon gives all its energy to the atom and changes angle


                                  2. inelastic scattering, the photon gives part of its energy to the atom and changes angle


                                  3. absorption, the photon gives all its energy to the absorbing atom/electron


                                  For all three, you can see that light both shows wave and particle characteristics. It is a misconception that the experiments about interaction are about the particle characteristics alone. Even in these three cases, when light interacts with matter, it behaves as a wave, in certain cases it changes angle, goes through slits, splits into partial waves, these partial waves interact with themselves, create interference patterns. Even in the case of absorption, when the photon ceises to exist, and transforms into the kinetic energy of the electron, it shows wave nature and particle both since the absorbing atomic system and electron both have wave and particle characteristics too.



                                  So there is no reason to really make a hard decision whether light would choose, it does not. It has both wave and particle characteristics as intrinsic, and it is up to us who prepare the experiment and analyze the data which we want to focus on.






                                  share|cite|improve this answer









                                  $endgroup$

















                                    1












                                    $begingroup$

                                    binaryfunt and Ben Crowell's answer is closest to the truth. Let me add a few things.



                                    You get confused because you read sentences like, light behaves as a wave when it travels in space and as a particle when it interferes with matter. That is not true. In reality light has characteristics of both waves and particles at the same time. The truth is, we do not know what actually light is, or what it is really made of.



                                    In reality, light is not what decides or behaves different ways, it is us, the ones preparing the experiments, who decide whether we will select from the observed (both wave or particle like) nature the more interesting wave or particle like characteristics, and analyze those for certain purposes.



                                    What we try to do, is model with our mathematical ways the real phenomenon in our physical world, light itself.



                                    The reason you get confused is because there are two ways to really model this phenomenon:



                                    1. classical EM waves


                                    2. QM particles, photons


                                    You would think they do not match, but that is not true. In reality the two methods come together perfectly, as we build up the classical EM waves from herds of QM photons.



                                    In our currently accepted theory, the Standard Model, we talk about elementary particles, that have no internal structure, nor spatial extent, and we call them point like particles. One of these particles is the photon. We call it the quanta of the EM wave.



                                    It is very important to see that the basic confusion lies in whether you want to analyze the wave or the particle characteristics of light in a certain experiment.



                                    We usually model light in two ways, and that is what confuses you, these to ways are whether it travels or when it interacts with matter. This is basically a misconception, because light could be analyzed to show wave and particles characteristics both when it travels and when it interacts with matter.



                                    It comes from a misconception to choose between travel and interaction, but for the sake of argument, lets see:



                                    1. travel

                                    You can use Maxwell's original equations or use QFT to treat photons and light as excitation of the EM field, that travels through space, and this propagation is modeled in our theories as a wave. This is because this wave model is what best fits the experimental data. Now we can show that the QM or QFT field excitation model of the photon propagating through space is the same fitting the data perfectly from the experiments.



                                    So you see light has both wave and particle characteristics even as it travels through space.



                                    1. interaction with matter

                                    Now when a photon interacts with an atom, three things can happen:



                                    1. elastic scattering, the photon gives all its energy to the atom and changes angle


                                    2. inelastic scattering, the photon gives part of its energy to the atom and changes angle


                                    3. absorption, the photon gives all its energy to the absorbing atom/electron


                                    For all three, you can see that light both shows wave and particle characteristics. It is a misconception that the experiments about interaction are about the particle characteristics alone. Even in these three cases, when light interacts with matter, it behaves as a wave, in certain cases it changes angle, goes through slits, splits into partial waves, these partial waves interact with themselves, create interference patterns. Even in the case of absorption, when the photon ceises to exist, and transforms into the kinetic energy of the electron, it shows wave nature and particle both since the absorbing atomic system and electron both have wave and particle characteristics too.



                                    So there is no reason to really make a hard decision whether light would choose, it does not. It has both wave and particle characteristics as intrinsic, and it is up to us who prepare the experiment and analyze the data which we want to focus on.






                                    share|cite|improve this answer









                                    $endgroup$















                                      1












                                      1








                                      1





                                      $begingroup$

                                      binaryfunt and Ben Crowell's answer is closest to the truth. Let me add a few things.



                                      You get confused because you read sentences like, light behaves as a wave when it travels in space and as a particle when it interferes with matter. That is not true. In reality light has characteristics of both waves and particles at the same time. The truth is, we do not know what actually light is, or what it is really made of.



                                      In reality, light is not what decides or behaves different ways, it is us, the ones preparing the experiments, who decide whether we will select from the observed (both wave or particle like) nature the more interesting wave or particle like characteristics, and analyze those for certain purposes.



                                      What we try to do, is model with our mathematical ways the real phenomenon in our physical world, light itself.



                                      The reason you get confused is because there are two ways to really model this phenomenon:



                                      1. classical EM waves


                                      2. QM particles, photons


                                      You would think they do not match, but that is not true. In reality the two methods come together perfectly, as we build up the classical EM waves from herds of QM photons.



                                      In our currently accepted theory, the Standard Model, we talk about elementary particles, that have no internal structure, nor spatial extent, and we call them point like particles. One of these particles is the photon. We call it the quanta of the EM wave.



                                      It is very important to see that the basic confusion lies in whether you want to analyze the wave or the particle characteristics of light in a certain experiment.



                                      We usually model light in two ways, and that is what confuses you, these to ways are whether it travels or when it interacts with matter. This is basically a misconception, because light could be analyzed to show wave and particles characteristics both when it travels and when it interacts with matter.



                                      It comes from a misconception to choose between travel and interaction, but for the sake of argument, lets see:



                                      1. travel

                                      You can use Maxwell's original equations or use QFT to treat photons and light as excitation of the EM field, that travels through space, and this propagation is modeled in our theories as a wave. This is because this wave model is what best fits the experimental data. Now we can show that the QM or QFT field excitation model of the photon propagating through space is the same fitting the data perfectly from the experiments.



                                      So you see light has both wave and particle characteristics even as it travels through space.



                                      1. interaction with matter

                                      Now when a photon interacts with an atom, three things can happen:



                                      1. elastic scattering, the photon gives all its energy to the atom and changes angle


                                      2. inelastic scattering, the photon gives part of its energy to the atom and changes angle


                                      3. absorption, the photon gives all its energy to the absorbing atom/electron


                                      For all three, you can see that light both shows wave and particle characteristics. It is a misconception that the experiments about interaction are about the particle characteristics alone. Even in these three cases, when light interacts with matter, it behaves as a wave, in certain cases it changes angle, goes through slits, splits into partial waves, these partial waves interact with themselves, create interference patterns. Even in the case of absorption, when the photon ceises to exist, and transforms into the kinetic energy of the electron, it shows wave nature and particle both since the absorbing atomic system and electron both have wave and particle characteristics too.



                                      So there is no reason to really make a hard decision whether light would choose, it does not. It has both wave and particle characteristics as intrinsic, and it is up to us who prepare the experiment and analyze the data which we want to focus on.






                                      share|cite|improve this answer









                                      $endgroup$



                                      binaryfunt and Ben Crowell's answer is closest to the truth. Let me add a few things.



                                      You get confused because you read sentences like, light behaves as a wave when it travels in space and as a particle when it interferes with matter. That is not true. In reality light has characteristics of both waves and particles at the same time. The truth is, we do not know what actually light is, or what it is really made of.



                                      In reality, light is not what decides or behaves different ways, it is us, the ones preparing the experiments, who decide whether we will select from the observed (both wave or particle like) nature the more interesting wave or particle like characteristics, and analyze those for certain purposes.



                                      What we try to do, is model with our mathematical ways the real phenomenon in our physical world, light itself.



                                      The reason you get confused is because there are two ways to really model this phenomenon:



                                      1. classical EM waves


                                      2. QM particles, photons


                                      You would think they do not match, but that is not true. In reality the two methods come together perfectly, as we build up the classical EM waves from herds of QM photons.



                                      In our currently accepted theory, the Standard Model, we talk about elementary particles, that have no internal structure, nor spatial extent, and we call them point like particles. One of these particles is the photon. We call it the quanta of the EM wave.



                                      It is very important to see that the basic confusion lies in whether you want to analyze the wave or the particle characteristics of light in a certain experiment.



                                      We usually model light in two ways, and that is what confuses you, these to ways are whether it travels or when it interacts with matter. This is basically a misconception, because light could be analyzed to show wave and particles characteristics both when it travels and when it interacts with matter.



                                      It comes from a misconception to choose between travel and interaction, but for the sake of argument, lets see:



                                      1. travel

                                      You can use Maxwell's original equations or use QFT to treat photons and light as excitation of the EM field, that travels through space, and this propagation is modeled in our theories as a wave. This is because this wave model is what best fits the experimental data. Now we can show that the QM or QFT field excitation model of the photon propagating through space is the same fitting the data perfectly from the experiments.



                                      So you see light has both wave and particle characteristics even as it travels through space.



                                      1. interaction with matter

                                      Now when a photon interacts with an atom, three things can happen:



                                      1. elastic scattering, the photon gives all its energy to the atom and changes angle


                                      2. inelastic scattering, the photon gives part of its energy to the atom and changes angle


                                      3. absorption, the photon gives all its energy to the absorbing atom/electron


                                      For all three, you can see that light both shows wave and particle characteristics. It is a misconception that the experiments about interaction are about the particle characteristics alone. Even in these three cases, when light interacts with matter, it behaves as a wave, in certain cases it changes angle, goes through slits, splits into partial waves, these partial waves interact with themselves, create interference patterns. Even in the case of absorption, when the photon ceises to exist, and transforms into the kinetic energy of the electron, it shows wave nature and particle both since the absorbing atomic system and electron both have wave and particle characteristics too.



                                      So there is no reason to really make a hard decision whether light would choose, it does not. It has both wave and particle characteristics as intrinsic, and it is up to us who prepare the experiment and analyze the data which we want to focus on.







                                      share|cite|improve this answer












                                      share|cite|improve this answer



                                      share|cite|improve this answer










                                      answered Apr 19 at 19:00









                                      Árpád SzendreiÁrpád Szendrei

                                      4,4411625




                                      4,4411625















                                          protected by ACuriousMind Apr 19 at 18:49



                                          Thank you for your interest in this question.
                                          Because it has attracted low-quality or spam answers that had to be removed, posting an answer now requires 10 reputation on this site (the association bonus does not count).



                                          Would you like to answer one of these unanswered questions instead?



                                          Popular posts from this blog

                                          Wikipedia:Vital articles Мазмуну Biography - Өмүр баян Philosophy and psychology - Философия жана психология Religion - Дин Social sciences - Коомдук илимдер Language and literature - Тил жана адабият Science - Илим Technology - Технология Arts and recreation - Искусство жана эс алуу History and geography - Тарых жана география Навигация менюсу

                                          Bruxelas-Capital Índice Historia | Composición | Situación lingüística | Clima | Cidades irmandadas | Notas | Véxase tamén | Menú de navegacióneO uso das linguas en Bruxelas e a situación do neerlandés"Rexión de Bruxelas Capital"o orixinalSitio da rexiónPáxina de Bruselas no sitio da Oficina de Promoción Turística de Valonia e BruxelasMapa Interactivo da Rexión de Bruxelas-CapitaleeWorldCat332144929079854441105155190212ID28008674080552-90000 0001 0666 3698n94104302ID540940339365017018237

                                          What should I write in an apology letter, since I have decided not to join a company after accepting an offer letterShould I keep looking after accepting a job offer?What should I do when I've been verbally told I would get an offer letter, but still haven't gotten one after 4 weeks?Do I accept an offer from a company that I am not likely to join?New job hasn't confirmed starting date and I want to give current employer as much notice as possibleHow should I address my manager in my resignation letter?HR delayed background verification, now jobless as resignedNo email communication after accepting a formal written offer. How should I phrase the call?What should I do if after receiving a verbal offer letter I am informed that my written job offer is put on hold due to some internal issues?Should I inform the current employer that I am about to resign within 1-2 weeks since I have signed the offer letter and waiting for visa?What company will do, if I send their offer letter to another company